FINANCE 3303 FINAL

Réussis tes devoirs et examens dès maintenant avec Quizwiz!

A discount bond that pays interest semiannually will: I. Have a lower price than an equivalent annual payment bond II. Have a higher EAR than an equivalent annual payment bond III. Sell for less than its conversion value A) I and II only B) I and III only C) II and III only D) I, II, and III

A

A dollar-denominated deposit at a London bank is called ________. A) eurodollars B) LIBOR C) fed funds D) bankers' acceptance

A

Ace Frisbee Corporation produces a good that is very mature in the firm's product life cycles. Ace Frisbee Corporation is expected to pay a dividend in year 1 of $3, a dividend in year 2 of $2, and a dividend in year 3 of $1. After year 3, dividends are expected to decline at the rate of 2% per year. An appropriate required return for the stock is 8%. Using the multistage DDM, the stock should be worth ________ today. A) $13.07 B) $13.58 C) $18.25 D) $18.78

A

Ace Ventura, Inc., has expected earnings of $5 per share for next year. The firm's ROE is 15%, and its earnings retention ratio is 40%. If the firm's market capitalization rate is 10%, what is the present value of its growth opportunities? A) $25 B) $50 C) $75 D) $100

A

All else the same, an American-style option will be ________ valuable than a ________ style option. A) more; European- B) less; European- C) more; Canadian- D) less; Canadian-

A

An American call option gives the buyer the right to ________. A) buy the underlying asset at the exercise price on or before the expiration date B) buy the underlying asset at the exercise price only at the expiration date C) sell the underlying asset at the exercise price on or before the expiration date D) sell the underlying asset at the exercise price only at the expiration date

A

Interior Airline is expected to pay a dividend of $3 in the upcoming year. Dividends are expected to grow at the rate of 10% per year. The risk-free rate of return is 4%, and the expected return on the market portfolio is 13%. The stock of Interior Airline has a beta of 1.4. Using the constant-growth DDM, the intrinsic value of the stock is ________. A) $45.45 B) $22.73 C) $27.78 D) $41.67

A

Money market securities are sometimes referred to as cash equivalents because ________. A) they are safe and marketable B) they are not liquid C) they are high-risk D) they are low-denomination

A

Most evidence indicates that U.S. stock markets are _______________________. A. reasonably weak-form and semistrong-form efficient B. strong-form efficient C. reasonably weak-form but not semistrong- or strong-form efficient D. neither weak-, semistrong-, nor strong-form efficient

A

On a given day a stock dealer maintains a bid price of $1,000.50 for a bond and an ask price of $1003.25. The dealer made 10 trades that totaled 500 bonds traded that day. What was the dealer's gross trading profit for this security? A) $1,375 B) $500 C) $275 D) $1,450

A

On a standard expected return versus standard deviation graph, investors will prefer portfolios that lie to the ________ the current investment opportunity set. A) left and above B) left and below C) right and above D) right and below

A

One can profit from an arbitrage opportunity by A) taking a long position in the cheaper market and a short position in the expensive market. B) taking a short position in the cheaper market and a long position in the expensive market. C) taking a long position in both markets. D) taking a short position in both markets.

A

Serial bonds are associated with ________. A) staggered maturity dates B) collateral C) coupon payment dates D) conversion features

A

Strips and straps are variations of ________. A) straddles B) collars C) money spreads D) time spreads

A

The Dow Jones Industrial Average is ________. A) a price-weighted average B) a value weight and average C) an equally weighted average D) an unweighted average

A

The New York Stock Exchange is a good example of ________. A) an auction market B) a brokered market C) a dealer market D) a direct search market

A

The ________ decision should take precedence over the ________ decision. A) asset allocation; stock selection B) bond selection; mutual fund selection C) stock selection; asset allocation D) stock selection; mutual fund selection

A

The free cash flow to the firm is reported as $198 million. The interest expense to the firm is $15 million. If the tax rate is 35% and the net debt of the firm increased by $20 million, what is the approximate market value of the firm if the FCFE grows at 3% and the cost of equity is 14%? A) $1,950 billion B) $2,497 billion C) $2,585 billion D) $3,098 billion

A

The free cash flow to the firm is reported as $275 million. The interest expense to the firm is $60 million. If the tax rate is 35% and the net debt of the firm increased by $33 million, what is the free cash flow to the equity holders of the firm? A) $269 million B) $296 million C) $305 million D) $327 million

A

The free cash flow to the firm is reported as $405 million. The interest expense to the firm is $76 million. If the tax rate is 35% and the net debt of the firm increased by $50 million, what is the free cash flow to the equity holders of the firm? A) $405.6 million B) $454.2 million C) $505.8 million D) $553.5 million

A

Value stocks are more likely to have a PEG ratio ________. A) less than 1 B) equal to 1 C) greater than 1 D) less than zero

A

You invest in the stock of Rayleigh Corp. and write a call option on Rayleigh Corp. This strategy is called a ________. A) covered call B) long straddle C) naked call D) money spread

A

A bond that has no collateral is called a ________. A) callable bond B) debenture C) junk bond D) mortgage

B

A stock's alpha measures the stock's ________. A) expected return B) abnormal return C) excess return D) residual return

B

An investor pays $989.40 for a bond. The bond has an annual coupon rate of 4.8%. What is the current yield on this bond? A) 4.8% B) 4.85% C) 9.6% D) 9.7%

B

Analysis of bond returns over a multiyear horizon based on forecasts of the bond's yield to maturity and reinvestment rate of coupons is called ________. A) multiyear analysis B) horizon analysis C) maturity analysis D) reinvestment analysis

B

Approximately ________ of trades involving shares issued by firms listed on the New York Stock Exchange actually take place on the New York Stock Exchange. A) 50% B) 25% C) 60% D) 75%

B

Choosing stocks by searching for predictable patterns in stock prices is called ________. A. fundamental analysis B. technical analysis C. index management D. random-walk investing

B

Deposits of commercial banks at the Federal Reserve are called ________. A) bankers' acceptances B) federal funds C) repurchase agreements D) time deposits

B

In 1973, trading of standardized options on a national exchange started on the ________. A) AMEX B) CBOE C) NYSE D) CFTC

B

In 2007, the NASDAQ stock market merged with ________. A) Euronext B) OMX, which operates seven Nordic and Baltic stock exchanges C) the International Securities Exchange (ISE) D) BATS

B

Longer-term American-style options with maturities of up to 3 years are called ________. A) warrants B) LEAPS C) GICs D) CATs

B

You short-sell 200 shares of Rock Creek Fly Fishing Co., now selling for $50 per share. If you want to limit your loss to $2,500, you should place a stop-buy order at ________. A) $37.50 B) $62.50 C) $56.25 D) $59.75

B

A ________ gives its holder the right to sell an asset for a specified exercise price on or before a specified expiration date. A) call option B) futures contract C) put option D) interest rate swap

C

A level ________ subscriber to the NASDAQ system may enter bid and ask prices. A) 1 B) 2 C) 3 D) 4

C

According to historical data, over the long run which of the following assets has the best chance to provide the best after-inflation, after-tax rate of return? A) long-term Treasury bonds B) corporate bonds C) common stocks D) preferred stocks

C

An adjusted beta will be ________ than the unadjusted beta. A) lower B) higher C) closer to 1 D) closer to 0

C

TIPS are an example of ________. A) Eurobonds B) convertible bonds C) indexed bonds D) catastrophe bonds

C

The Standard & Poor's 500 is ________ weighted index. A) an equally B) a price- C) a value- D) a share-

C

Which of the following is not a money market instrument? A) Treasury bill B) commercial paper C) preferred stock D) bankers' acceptance

C

) The issuer of ________ bond may choose to pay interest either in cash or in additional bonds. A) an asset-backed B) a TIPS C) a catastrophe D) a pay-in-kind

D

According to the CAPM, what is the market risk premium given an expected return on a security of 13.6%, a stock beta of 1.2, and a risk-free interest rate of 4%? A) 4% B) 4.8% C) 6.6% D) 8%

D

According to the liquidity preference theory of the term structure of interest rates, an increase in the yield on long-term corporate bonds versus short-term bonds could be due to ________. A) declining liquidity premiums B) an expectation of an upcoming recession C) a decline in future inflation expectations D) an increase in expected interest rate volatility

D

In a well-diversified portfolio, ________ risk is negligible. A) nondiversifiable B) market C) systematic D) unsystematic

D

Level 3 NASDAQ subscribers ________. A) are registered market makers B) can post bid and ask prices C) have the fastest execution of trades D) all of these options

D

The potential loss for a writer of a naked call option on a stock is ________. A) equal to the call premium B) larger the lower the stock price C) limited D) unlimited

D

To construct a riskless portfolio using two risky stocks, one would need to find two stocks with a correlation coefficient of ________. A) -0.5 B) 0.0 C) 0.5 D) -1.0

D

Which of the following possible provisions of a bond indenture is designed to ease the burden of principal repayment by spreading it out over several years? A) callable feature B) convertible feature C) subordination clause D) sinking fund

D

Which one of the following is not an example of a brokered market? A) residential real estate market B) market for large block security transactions C) primary market for securities D) NASDAQ

D

________ often accompany short sales and are used to limit potential losses from the short position. A) Limit orders B) Restricted orders C) Limit loss orders D) Stop-buy orders

D

"Buy a stock if its price moves up by 2% more than the Dow Average" is an example of a _________________. A. trading rule B. market anomaly C. fundamental approach D. passive trading strategy

A

A ________ bond gives the issuer an option to retire the bond before maturity at a specific price after a specific date. A) callable B) coupon C) puttable D) Treasury

A

A ________ gives its holder the right to buy an asset for a specified exercise price on or before a specified expiration date. A) call option B) futures contract C) put option D) interest rate swap

A

A bond has a 5% coupon rate. The coupon is paid semiannually, and the last coupon was paid 35 days ago. If the bond has a par value of $1,000, what is the accrued interest? A) $4.81 B) $14.24 C) $25 D) $50

A

A callable bond pays annual interest of $60, has a par value of $1,000, matures in 20 years but is callable in 10 years at a price of $1,100, and has a value today of $1055.84. The yield to call on this bond is ________. A) 6% B) 6.58% C) 7.2% D) 8%

A

A collateral trust bond is ________. A) secured by other securities held by the firm B) secured by equipment owned by the firm C) secured by property owned by the firm D) unsecured

A

A firm has a stock price of $54.75 per share. The firm's earnings are $75 million, and the firm has 20 million shares outstanding. The firm has an ROE of 15% and a plowback of 65%. What is the firm's PEG ratio? A) 1.5 B) 1.25 C) 1.1 D) 1

A

A firm has current assets that could be sold for their book value of $10 million. The book value of its fixed assets is $60 million, but they could be sold for $95 million today. The firm has total debt at a book value of $40 million, but interest rate changes have increased the value of the debt to a current market value of $50 million. This firm's market-to-book ratio is ________. A) 1.83 B) 1.5 C) 1.35 D) 1.46

A

A firm that has an ROE of 12% is considering cutting its dividend payout. The stockholders of the firm desire a dividend yield of 4% and a capital gain yield of 9%. Given this information, which of the following statements is (are) correct? I. All else equal, the firm's growth rate will accelerate after the payout change. II. All else equal, the firm's stock price will go up after the payout change. III. All else equal, the firm's P/E ratio will increase after the payout change. A) I only B) I and II only C) II and III only D) I, II, and III

A

A portfolio is composed of two stocks, A and B. Stock A has a standard deviation of return of 24%, while stock B has a standard deviation of return of 18%. Stock A comprises 60% of the portfolio, while stock B comprises 40% of the portfolio. If the variance of return on the portfolio is .0380, the correlation coefficient between the returns on A and B is ________. A) .583 B) .225 C) .327 D) .128

A

A put on Sanders stock with a strike price of $35 is priced at $2 per share, while a call with a strike price of $35 is priced at $3.50. The maximum per-share loss to the writer of an uncovered put is ________, and the maximum per-share gain to the writer of an uncovered call is ________. A) $33; $3.50 B) $33; $31.50 C) $35; $3.50 D) $35; $35

A

A stock has a beta of 1.3. The systematic risk of this stock is ________ the stock market as a whole. A) higher than B) lower than C) equal to D) indeterminable compared to

A

A stock is priced at $45 per share. The stock has earnings per share of $3 and a market capitalization rate of 14%. What is the stock's PVGO? A) $23.57 B) $15 C) $19.78 D) $21.34

A

A technical analyst is most likely to be affiliated with which investment philosophy? A. Active management B. Buy and hold C. Passive investment D. Index funds

A

A writer of a call option will want the value of the underlying asset to ________, and a buyer of a put option will want the value of the underlying asset to ________. A) decrease; decrease B) decrease; increase C) increase; decrease D) increase; increase

A

A zero-coupon bond has a yield to maturity of 5% and a par value of $1,000. If the bond matures in 16 years, it should sell for a price of ________ today. A) $458.11 B) $641.11 C) $789.11 D) $1,100.11

A

An index computed from a simple average of returns is a/an ________. A) equal weighted index B) value weighted index C) price weighted index D) share weighted index

A

An individual who goes short in a futures position ________. A) commits to delivering the underlying commodity at contract maturity B) commits to purchasing the underlying commodity at contract maturity C) has the right to deliver the underlying commodity at contract maturity D) has the right to purchase the underlying commodity at contract maturity

A

An investor can design a risky portfolio based on two stocks, A and B. Stock A has an expected return of 18% and a standard deviation of return of 20%. Stock B has an expected return of 14% and a standard deviation of return of 5%. The correlation coefficient between the returns of A and B is .50. The risk-free rate of return is 10%. The expected return on the optimal risky portfolio is ________. A) 14% B) 15.6% C) 16.4% D) 18%

A

An investor can design a risky portfolio based on two stocks, A and B. Stock A has an expected return of 18% and a standard deviation of return of 20%. Stock B has an expected return of 14% and a standard deviation of return of 5%. The correlation coefficient between the returns of A and B is .50. The risk-free rate of return is 10%. The proportion of the optimal risky portfolio that should be invested in stock A is ________. A) 0% B) 40% C) 60% D) 100%

A

An investor is bearish on a particular stock and decided to buy a put with a strike price of $25. Ignoring commissions, if the option was purchased for a price of $.85, what is the break-even point for the investor? A) $24.15 B) $25 C) $25.87 D) $27.86

A

An investor puts up $5,000 but borrows an equal amount of money from his broker to double the amount invested to $10,000. The broker charges 7% on the loan. The stock was originally purchased at $25 per share, and in 1 year the investor sells the stock for $28. The investor's rate of return was ________. A) 17% B) 12% C) 14% D) 19%

A

Annie's Donut Shops, Inc., has expected earnings of $3 per share for next year. The firm's ROE is 18%, and its earnings retention ratio is 60%. If the firm's market capitalization rate is 12%, what is the value of the firm excluding any growth opportunities? A) $25 B) $50 C) $83.33 D) $208

A

Annual percentage rates can be converted to effective annual rates by means of the following formula: A) [1 + (APR/n)]n - 1 B) (APR)(n) C) (APR/n) D) (periodic rate)(n)

A

Arbitrage is based on the idea that ________. A) assets with identical risks must have the same expected rate of return B) securities with similar risk should sell at different prices C) the expected returns from equally risky assets are different D) markets are perfectly efficient

A

As a result of flash crashes, the SEC is trying circuit breakers that will halt trading for 5 minutes if large stocks' prices change by more than ________ in a 5-minute period. A) 10% B) 20% C) 30% D) 40%

A

Asset A has an expected return of 15% and a reward-to-variability ratio of .4. Asset B has an expected return of 20% and a reward-to-variability ratio of .3. A risk-averse investor would prefer a portfolio using the risk-free asset and ________. A) asset A B) asset B C) no risky asset D) The answer cannot be determined from the data given.

A

Asset A has an expected return of 20% and a standard deviation of 25%. The risk-free rate is 10%. What is the reward-to-variability ratio? A) .40 B) .50 C) .75 D) .80

A

Assume that both X and Y are well-diversified portfolios and the risk-free rate is 8%. Portfolio X has an expected return of 14% and a beta of 1. Portfolio Y has an expected return of 9.5% and a beta of .25. In this situation, you would conclude that portfolios X and Y ________. A) are in equilibrium B) offer an arbitrage opportunity C) are both underpriced D) are both fairly priced

A

Assuming semiannual compounding, a 20-year zero coupon bond with a par value of $1,000 and a required return of 12% would be priced at ________. A) $97.22 B) $104.49 C) $364.08 D) $732.14

A

At contract maturity the value of a call option is ________, where X equals the option's strike price and ST is the stock price at contract expiration. A) max (0, ST − X) B) min (0, ST − X) C) max (0, X − ST) D) min (0, X − ST)

A

At expiration of an option contract, which phrase describes the point at which both calls and puts have the same gross profit? A) at the money B) in the money C) out of the money D) knocked in

A

Bill Jones inherited 5,000 shares of stock priced at $45 per share. He does not want to sell the stock this year due to tax reasons, but he is concerned that the stock will drop in value before year-end. Bill wants to use a collar to ensure that he minimizes his risk and doesn't incur too much cost in deferring the gain. January call options with a strike of $50 are quoted at a cost of $2, and January puts with a $40 exercise price are quoted at a cost of $3. If Bill establishes the collar and the stock price winds up at $35 in January, Bill's net position value including the option profit or loss and the stock is ________. A) $195,000 B) $220,000 C) $175,000 D) $215,000

A

Bonds issued in the currency of the issuer's country but sold in other national markets are called ________. A) Eurobonds B) Yankee bonds C) Samurai bonds D) foreign bonds

A

Caribou Gold Mining Corporation is expected to pay a dividend of $6 in the upcoming year. Dividends are expected to decline at the rate of 3% per year. The risk-free rate of return is 5%, and the expected return on the market portfolio is 13%. The stock of Caribou Gold Mining Corporation has a beta of 0.5. Using the constant-growth DDM, the intrinsic value of the stock is ________. A) $50 B) $100 C) $150 D) $200

A

Consider a 7-year bond with a 9% coupon and a yield to maturity of 12%. If interest rates remain constant, 1 year from now the price of this bond will be ________. A) higher B) lower C) the same D) indeterminate

A

Consider a Treasury bill with a rate of return of 5% and the following risky securities: Security A: E(r) = .15; variance = .0400 Security B: E(r) = .10; variance = .0225 Security C: E(r) = .12; variance = .1000 Security D: E(r) = .13; variance = .0625 The investor must develop a complete portfolio by combining the risk-free asset with one of the securities mentioned above. The security the investor should choose as part of her complete portfolio to achieve the best CAL would be ________. A) security A B) security B C) security C D) security D

A

Consider the following $1,000 par value zero-coupon bonds: Bond A/B/C/D/E Years to Maturity 1/2/3/4/5 Yield to Maturity 6.00%/7.00%/7.99%/9.41%/10.70% The expected 1-year interest rate 4 years from now should be ________. A) 16% B) 18% C) 20% D) 22%

A

Consider the one-factor APT. The variance of the return on the factor portfolio is .08. The beta of a well-diversified portfolio on the factor is 1.2. The variance of the return on the well-diversified portfolio is approximately ________. A) .1152 B) .1270 C) .1521 D) .1342

A

During the 1926-2013 period the geometric mean return on Treasury bonds was ________. A) 5.07% B) 5.56% C) 9.34% D) 11.43%

A

Each of two stocks, A and B, is expected to pay a dividend of $7 in the upcoming year. The expected growth rate of dividends is 6% for both stocks. You require a return of 10% on stock A and a return of 12% on stock B. Using the constant-growth DDM, the intrinsic value of stock A ________. A) will be higher than the intrinsic value of stock B B) will be the same as the intrinsic value of stock B C) will be less than the intrinsic value of stock B D) The answer cannot be determined from the information given.

A

Eagle Brand Arrowheads has expected earnings of $1.25 per share and a market capitalization rate of 12%. Earnings are expected to grow at 5% per year indefinitely. The firm has a 40% plowback ratio. By how much does the firm's ROE exceed the market capitalization rate? A) .5% B) 1% C) 1.5% D) 2%

A

Earnings yields tend to ________ when Treasury yields fall. A) fall B) rise C) remain unchanged D) fluctuate wildly

A

Estimates of a stock's intrinsic value calculated with the free cash flow methodology depend most critically on ________. A) the terminal value used B) whether one uses FCFF or FCFE C) the time period used to estimate the cash flows D) whether the firm is currently paying dividends

A

Eurodollars are ________. A) dollar-denominated deposits at any foreign bank or foreign branch of an American bank B) dollar-denominated bonds issued by firms outside their home market C) currency issued by Euro Disney and traded in France D) dollars that wind up in banks as a result of money-laundering activities

A

Firms with higher expected growth rates tend to have P/E ratios that are ________ the P/E ratios of firms with lower expected growth rates. A) higher than B) equal to C) lower than D) There is not necessarily any linkage between risk and P/E ratios.

A

Flanders, Inc., has expected earnings of $4 per share for next year. The firm's ROE is 8%, and its earnings retention ratio is 40%. If the firm's market capitalization rate is 15%, what is the present value of its growth opportunities? A) −$6.33 B) $0 C) $20.34 D) $26.67

A

Historically, the best asset for the long-term investor wanting to fend off the threats of inflation and taxes while making his money grow has been ________. A) stocks B) bonds C) money market funds D) Treasury bills

A

If an investor does not diversify his portfolio and instead puts all of his money in one stock, the appropriate measure of security risk for that investor is the ________. A) stock's standard deviation B) variance of the market C) stock's beta D) covariance with the market index

A

If an investor uses the full amount of margin available, the equity in a margin account used for a stock purchase can be found as ________. A) market value of the stock - amount owed on the margin loan B) market value of the stock + amount owed on the margin loan C) market value of the stock ÷ margin loan D) margin loan × market value of the stock

A

If enough investors decide to purchase stocks, they are likely to drive up stock prices, thereby causing ________ and ________. A) expected returns to fall; risk premiums to fall B) expected returns to rise; risk premiums to fall C) expected returns to rise; risk premiums to rise D) expected returns to fall; risk premiums to rise

A

If the U.S. capital markets are not informationally efficient, ______. A. the markets cannot be allocationally efficient B. systematic risk does not matter C. no type of analysis can be used to generate abnormal returns D. returns must follow a random walk

A

If the coupon rate on a bond is 4.5% and the bond is selling at a premium, which of the following is the most likely yield to maturity on the bond? A) 4.3% B) 4.5% C) 5.2% D) 5.5%

A

If the nominal rate of return on investment is 6% and inflation is 2% over a holding period, what is the real rate of return on this investment? A) 3.92% B) 4% C) 4.12% D) 6%

A

If you believe in the __________ form of the EMH, you believe that stock prices reflect all publicly available information but not information that is available only to insiders. A. semistrong B. strong C. weak D. perfect

A

If you thought prices of stock would be rising over the next few months, you might want to ________ on the stock. A) purchase a call option B) purchase a put option C) sell a futures contract D) place a short-sale order

A

In 2014, BATS advertised average latency times of approximately ________. A) 100 microseconds B) 200 microseconds C) 1 second D) 5 seconds

A

In a ________ index, changes in the value of the stock with the greatest market value will move the index value the most, everything else equal. A) value-weighted index B) equally weighted index C) price-weighted index D) bond price index

A

In a single-factor market model the beta of a stock ________. A) measures the stock's contribution to the standard deviation of the market portfolio B) measures the stock's unsystematic risk C) changes with the variance of the residuals D) measures the stock's contribution to the standard deviation of the stock

A

In his famous critique of the CAPM, Roll argued that the CAPM ________. A) is not testable because the true market portfolio can never be observed B) is of limited use because systematic risk can never be entirely eliminated C) should be replaced by the APT D) should be replaced by the Fama-French three-factor model

A

In regard to bonds, convexity relates to the ________. A) shape of the bond price curve with respect to interest rates B) shape of the yield curve with respect to maturity C) slope of the yield curve with respect to liquidity premiums D) size of the bid-ask spread

A

In the mean standard deviation graph, the line that connects the risk-free rate and the optimal risky portfolio, P, is called the ________. A) capital allocation line B) indifference curve C) investor's utility line D) security market line

A

Liquidity is a risk factor that ________. A) has yet to be accurately measured and incorporated into portfolio management B) is unaffected by trading mechanisms on various stock exchanges C) has no effect on the market value of an asset D) affects bond prices but not stock prices

A

Maintenance requirements for margin accounts are set by ________. A) brokerage firms B) the SEC C) the Federal Reserve System's Board of Governors D) the Supreme Court

A

One type of passive portfolio management is ________. A. investing in a well-diversified portfolio without attempting to search out mispriced securities B. investing in a well-diversified portfolio while only seeking out passively mispriced securities C. investing an equal dollar amount in index stocks D. investing in an equal amount of shares in each of the index stocks

A

Ownership of a call option entitles the owner to the ________ to ________ a specific stock, on or before a specific date, at a specific price. A) right; buy B) right; sell C) obligation; buy D) obligation; sell

A

Private placements can be advantageous, compared to public issue, because: I. Private placements are cheaper to market than public issues. II. Private placements may still be sold to the general public under SEC Rule 144A. III. Privately placed securities trade on secondary markets. A) I only B) I and III only C) II and III only D) I, II, and III

A

Research has revealed that regardless of what the current estimate of a firm's beta is, beta will tend to move closer to ________ over time. A) 1 B) 0 C) -1 D) .5

A

Standard deviation of portfolio returns is a measure of ________. A) total risk B) relative systematic risk C) relative nonsystematic risk D) relative business risk

A

Suppose you purchase one Texas Insurance August 75 call contract quoted at $8.50 and write one Texas Insurance August 80 call contract quoted at $6. If, at expiration, the price of a share of Texas Instruments stock is $79, your profit would be ________. A) $150 B) $400 C) $600 D) $1,850

A

The NYSE acquired the ECN ________, and NASDAQ recently acquired the ECN ________. A) Archipelago; Instinet B) Instinet; Archipelago C) Island; Instinet D) LSE; Euronext

A

The NYSE has lost market share to ECNs in recent years. Part of the NYSE's response to the growth of ECNs has been to: I. Purchase Archipelago, a major ECN, and rename it NYSE Arca II. Enable automatic trade execution through its new Market Center III. Impose a tighter limit on bid-ask spreads A) I only B) II and III only C) I and II only D) I, II, and III

A

The Option Clearing Corporation is owned by ________. A) the exchanges on which stock options are traded B) the Federal Deposit Insurance Corporation C) the Federal Reserve System D) major U.S. banks

A

The ________ is the document that defines the contract between the bond issuer and the bondholder. A) indenture B) covenant agreement C) trustee agreement D) collateral statement

A

The ________ price is the price at which a dealer is willing to purchase a security. A) bid B) ask C) clearing D) settlement

A

The ________ the ratio of municipal bond yields to corporate bond yields, the ________ the cutoff tax bracket at which more individuals will prefer to hold municipal debt.

A

The accounting measure of a firm's equity value generated by applying accounting principles to asset and liability acquisitions is called ________. A) book value B) market value C) liquidation value D) Tobin's q

A

The bulk of most initial public offerings (IPOs) of equity securities goes to ________. A) institutional investors B) individual investors C) the firm's current shareholders D) day traders

A

The common stock of the Avalon Corporation has been trading in a narrow range around $40 per share for months, and you believe it is going to stay in that range for the next 3 months. The price of a 3-month put option with an exercise price of $40 is $3, and a call with the same expiration date and exercise price sells for $4. How can you create a position involving a put, a call, and riskless lending that would have the same payoff structure as the stock at expiration? A) Buy the call, sell the put; lend the present value of $40. B) Sell the call, buy the put; lend the present value of $40. C) Buy the call, sell the put; borrow the present value of $40. D) Sell the call, buy the put; borrow the present value of $40.

A

The formula is used to calculate the ________. A) Sharpe ratio B) Treynor measure C) coefficient of variation D) real rate of return

A

The initial maturities of most exchange-traded options are generally ________. A) less than 1 year B) less than 2 years C) between 1 and 2 years D) between 1 and 3 years

A

The inside quotes on a limit order book can be found ________. A) at the top of the list B) at the bottom of the list C) by taking the averages of the bid and ask prices on the list D) only by direct contact with the specialist who maintains the book

A

The invoice price of a bond is the ________. A) stated or flat price in a quote sheet plus accrued interest B) stated or flat price in a quote sheet minus accrued interest C) bid price D) average of the bid and ask price

A

The market risk premium is defined as ________. A) the difference between the return on an index fund and the return on Treasury bills B) the difference between the return on a small-firm mutual fund and the return on the Standard & Poor's 500 Index C) the difference between the return on the risky asset with the lowest returns and the return on Treasury bills D) the difference between the return on the highest-yielding asset and the return on the lowest-yielding asset

A

The maximum loss a buyer of a stock call option can suffer is the ________. A) call premium B) stock price C) stock price minus the value of the call D) strike price minus the stock price

A

The maximum maturity on commercial paper is ________. A) 270 days B) 180 days C) 90 days D) 30 days

A

The measure of unsystematic risk can be found from an index model as ________. A) residual standard deviation B) R-square C) degrees of freedom D) sum of squares of the regression

A

The most marketable money market security is ________. A) Treasury bills B) bankers' acceptances C) certificates of deposit D) common stock

A

The normal distribution is completely described by its ________. A) mean and standard deviation B) mean C) mode and standard deviation D) median and variance

A

The price of a bond (with par value of $1,000) at the beginning of a period is $980 and at the end of the period is $975. What is the holding-period return if the annual coupon rate is 4.5%? A) 4.08% B) 4.5% C) 5.1% D) 5.6%

A

The price on a Treasury bond is 104.3625, with a yield to maturity of 3.45%. The price on a comparable maturity corporate bond is 103.75, with a yield to maturity of 4.59%. What is the approximate percentage value of the credit risk of the corporate bond? A) 1.14% B) 3.45% C) 4.59% D) 8.04%

A

The price-to-sales ratio is probably most useful for firms in which phase of the industry life cycle? A) start-up phase B) consolidation C) maturity D) relative decline

A

The primary difference between Treasury notes and bonds is ________. A) maturity at issue B) default risk C) coupon rate D) tax status

A

The reward-to-volatility ratio is given by ________. A) the slope of the capital allocation line B) the second derivative of the capital allocation line C) the point at which the second derivative of the investor's indifference curve reaches zero D) the portfolio's excess return

A

The risk premium for exposure to exchange rates is 5%, and the firm has a beta relative to exchange rates of .4. The risk premium for exposure to the consumer price index is -6%, and the firm has a beta relative to the CPI of .8. If the risk-free rate is 3%, what is the expected return on this stock? A) .2% B) 1.5% C) 3.6% D) 4%

A

The term complete portfolio refers to a portfolio consisting of ________. A) the risk-free asset combined with at least one risky asset B) the market portfolio combined with the minimum-variance portfolio C) securities from domestic markets combined with securities from foreign markets D) common stocks combined with bonds

A

The value of a listed put option on a stock is lower when: I. The exercise price is higher. II. The contract approaches maturity. III. The stock decreases in value. IV. A stock split occurs. A) II only B) II and IV only C) I, II, and III only D) I, II, III, and IV

A

The weak form of the EMH states that ________ must be reflected in the current stock price. A. all past information, including security price and volume data B. all publicly available information C. all information, including inside information D. all costless information

A

The yield to maturity of a 10-year zero-coupon bond with a par value of $1,000 and a market price of $625 is ________. A) 4.8% B) 6.1% C) 7.7% D) 10.4%

A

Two investment advisers are comparing performance. Adviser A averaged a 20% return with a portfolio beta of 1.5, and adviser B averaged a 15% return with a portfolio beta of 1.2. If the T-bill rate was 5% and the market return during the period was 13%, which adviser was the better stock picker? A) Advisor A was better because he generated a larger alpha. B) Advisor B was better because she generated a larger alpha. C) Advisor A was better because he generated a higher return. D) Advisor B was better because she achieved a good return with a lower beta.

A

Under the pure expectations hypothesis and constant real interest rates for different maturities, an upward-sloping yield curve would indicate ________. A) expected increases in inflation over time B) expected decreases in inflation over time C) the presence of a liquidity premium D) that the equilibrium interest rate in the short-term part of the market is lower than the equilibrium interest rate in the long-term part of the market

A

What combination of puts and calls can simulate a long stock investment? A) long call and short put B) long call and long put C) short call and short put D) short call and long put

A

What is the standard deviation of a portfolio of two stocks given the following data: Stock A has a standard deviation of 18%. Stock B has a standard deviation of 14%. The portfolio contains 40% of stock A, and the correlation coefficient between the two stocks is -.23. A) 9.7% B) 12.2% C) 14% D) 15.6%

A

What is the tax exempt equivalent yield on a 9% bond yield given a marginal tax rate of 28%? A) 6.48% B) 7.25% C) 8.02% D) 9%

A

What strategy is designed to ensure a value within the bounds of two different stock prices? A) collar B) covered Call C) protective put D) straddle

A

What was the result of high-frequency traders' leaving the market during the flash crash of 2010? A) Market liquidity decreased. B) Market liquidity increased. C) Market volatility decreased. D) Trading frequency increased.

A

When a firm decides to sell securities it must first ensure ________. A) the preliminary registration statement is approved by the SEC B) the IPO is complete C) the offering is seasoned D) the lockup period expires

A

Which of the following does not approximate the performance of a buy-and-hold portfolio strategy? A) an equally weighted index B) a price-weighted index C) a value-weighted index D) all of these options (Weights are not a factor in this situation.)

A

Which of the following expressions represents the value of a call option to its holder on the expiration date? A) ST − X if ST > X, 0 if ST ≤ X B) − (ST − X) if ST > X, 0 if ST ≤ X C) 0 if ST ≥ X, X − ST if ST < X D) 0 if ST ≥ X, − (X − ST) if ST < X

A

Which of the following yield curves generally implies a normal healthy economy? A) positive slope B) negative slope C) flat D) hump-shaped curve

A

Which one of the following is a correct statement? A) Exercise of warrants results in more outstanding shares of stock, while exercise of listed call options does not. B) A convertible bond consists of a straight bond plus a specified number of detachable warrants. C) Call options always have an initial maturity greater than 1 year, while warrants have an initial maturity less than 1 year. D) Call options may be convertible into the stock, while warrants are not convertible into the stock.

A

Which one of the following is equal to the ratio of common shareholders' equity to common shares outstanding? A) book value per share B) liquidation value per share C) market value per share D) Tobin's q

A

Which one of the following measures time-weighted returns and allows for compounding? A) geometric average return B) arithmetic average return C) dollar-weighted return D) historical average return

A

Which strategy benefits from upside price movement and has some protection should the price of the security fall? A) Bull spread B) Long put C) Short call D) Straddle

A

Why is the holder of an option not required to post margin under the Option Clearing Corporation rules? A) Once an option is purchased, no further money is at risk. B) The seller pays all costs. C) The credit worthiness of the holder covers all potential losses. D) The holder must post securities instead of margin.

A

Yields on municipal bonds are typically ________ yields on corporate bonds of similar risk and time to maturity. A) lower than B) slightly higher than C) identical to D) twice as high as

A

You are cautiously bullish on the common stock of the Wildwood Corporation over the next several months. The current price of the stock is $50 per share. You want to establish a bullish money spread to help limit the cost of your option position. You find the following option quotes: Wildwood Corp Underlying Stock price: $50.00 Expiration Strike Call Put June 45.00 8.50 2.00 June 50.00 4.50 3.00 June 55.00 2.00 7.50 To establish a bull money spread with puts, you would ________. A) sell the 55 put and buy the 45 put B) buy the 45 put and buy the 55 put C) buy the 55 put and sell the 45 put D) sell the 45 put and sell the 55 put

A

You are considering investing $1,000 in a complete portfolio. The complete portfolio is composed of Treasury bills that pay 5% and a risky portfolio, P, constructed with two risky securities, X and Y. The optimal weights of X and Y in P are 60% and 40%, respectively. X has an expected rate of return of 14%, and Y has an expected rate of return of 10%. To form a complete portfolio with an expected rate of return of 11%, you should invest ________ of your complete portfolio in Treasury bills. A) 19% B) 25% C) 36% D) 50%

A

You buy a call option on Summit Corp. with an exercise price of $40 and an expiration date in September, and you write a call option on Summit Corp. with an exercise price of $40 and an expiration date in October. This strategy is called a ________. A) time spread B) long straddle C) short straddle D) money spread

A

You buy an 8-year $1,000 par value bond today that has a 6% yield and a 6% annual payment coupon. In 1 year promised yields have risen to 7%. Your 1-year holding-period return was ________. A) .61% B) −5.39% C) 1.28% D) −3.25%

A

You can be sure that a bond will sell at a premium to par when ________. A) its coupon rate is greater than its yield to maturity B) its coupon rate is less than its yield to maturity C) its coupon rate is equal to its yield to maturity D) its coupon rate is less than its conversion value

A

You consider buying a share of stock at a price of $25. The stock is expected to pay a dividend of $1.50 next year, and your advisory service tells you that you can expect to sell the stock in 1 year for $28. The stock's beta is 1.1, rf is 6%, and E[rm] = 16%. What is the stock's abnormal return? A) 1% B) 2% C) -1% D) -2%

A

You have a $50,000 portfolio consisting of Intel, GE, and Con Edison. You put $20,000 in Intel, $12,000 in GE, and the rest in Con Edison. Intel, GE, and Con Edison have betas of 1.3, 1, and .8, respectively. What is your portfolio beta?

A

You invest $1,000 in a complete portfolio. The complete portfolio is composed of a risky asset with an expected rate of return of 16% and a standard deviation of 20% and a Treasury bill with a rate of return of 6%. A portfolio that has an expected value in 1 year of $1,100 could be formed if you ________. A) place 40% of your money in the risky portfolio and the rest in the risk-free asset B) place 55% of your money in the risky portfolio and the rest in the risk-free asset C) place 60% of your money in the risky portfolio and the rest in the risk-free asset D) place 75% of your money in the risky portfolio and the rest in the risk-free asset

A

You invest $10,000 in a complete portfolio. The complete portfolio is composed of a risky asset with an expected rate of return of 15% and a standard deviation of 21% and a Treasury bill with a rate of return of 5%. How much money should be invested in the risky asset to form a portfolio with an expected return of 11%? A) $6,000 B) $4,000 C) $7,000 D) $3,000

A

You purchase a call option on a stock. The profit at contract maturity of the option position is ________, where X equals the option's strike price, ST is the stock price at contract expiration, and C0 is the original purchase price of the option. A) max (−C0, ST − X − C0) B) min (−C0, ST − X − C0) C) max (C0, ST − X + C0) D) max (0, ST − X − C0)

A

You sell short 200 shares of Doggie Treats Inc. that are currently selling at $25 per share. You post the 50% margin required on the short sale. If your broker requires a 30% maintenance margin, at what stock price will you get a margin call? (You earn no interest on the funds in your margin account, and the firm does not pay any dividends.) A) $28.85 B) $35.71 C) $31.50 D) $32.25

A

You sold short 300 shares of common stock at $30 per share. The initial margin is 50%. You must put up ________. A) $4,500 B) $6,000 C) $9,000 D) $10,000

A

You want to earn a return of 10% on each of two stocks, A and B. Each of the stocks is expected to pay a dividend of $4 in the upcoming year. The expected growth rate of dividends is 6% for stock A and 5% for stock B. Using the constant-growth DDM, the intrinsic value of stock A ________. A) will be higher than the intrinsic value of stock B B) will be the same as the intrinsic value of stock B C) will be less than the intrinsic value of stock B D) The answer cannot be determined from the information given.

A

You want to earn a return of 11% on each of two stocks, A and B. Stock A is expected to pay a dividend of $3 in the upcoming year, while stock B is expected to pay a dividend of $2 in the upcoming year. The expected growth rate of dividends for both stocks is 4%. Using the constant-growth DDM, the intrinsic value of stock A ________. A) will be higher than the intrinsic value of stock B B) will be the same as the intrinsic value of stock B C) will be less than the intrinsic value of stock B D) The answer cannot be determined from the information given.

A

You write one MBI July 120 call contract (equaling 100 shares) for a premium of $4. You hold the option until the expiration date, when MBI stock sells for $121 per share. You will realize a ________ on the investment. A) $300 profit B) $200 loss C) $600 loss D) $200 profit

A

Your investment has a 40% chance of earning a 15% rate of return, a 50% chance of earning a 10% rate of return, and a 10% chance of losing 3%. What is the standard deviation of this investment? A) 5.14% B) 7.59% C) 9.29% D) 8.43%

A

Your timing was good last year. You invested more in your portfolio right before prices went up, and you sold right before prices went down. In calculating historical performance measures, which one of the following will be the largest? A) dollar-weighted return B) geometric average return C) arithmetic average return D) mean holding-period return

A

$1,000 par value zero-coupon bonds (ignore liquidity premiums) Bond A/B/C/D/E Years to Maturity 1/2/3/4/5 Yield to Maturity 6.00%/7.50%/7.99%/8.49%/10.70% One year from now bond C should sell for ________ (to the nearest dollar). A) $857 B) $894 C) $835 D) $821

B

A 1% decline in yield will have the least effect on the price of a bond with a ________. A) 10-year maturity, selling at 80 B) 10-year maturity, selling at 100 C) 20-year maturity, selling at 80 D) 20-year maturity, selling at 100

B

A 6% coupon U.S. Treasury note pays interest on May 31 and November 30 and is traded for settlement on August 10. The accrued interest on the $100,000 face amount of this note is ________. A) $581.97 B) $1,170.33 C) $2,327.87 D) $3,000

B

A European call option gives the buyer the right to ________. A) buy the underlying asset at the exercise price on or before the expiration date B) buy the underlying asset at the exercise price only at the expiration date C) sell the underlying asset at the exercise price on or before the expiration date D) sell the underlying asset at the exercise price only at the expiration date

B

A T-bill quote sheet has 90-day T-bill quotes with a 4.92 ask and a 4.86 bid. If the bill has a $10,000 face value, an investor could sell this bill for ________. A) $10,000 B) $9,878.50 C) $9,877 D) $9,880.16

B

A bond has a flat price of $985, and it pays an annual coupon. The last coupon payment was made 90 days ago. What is the invoice price if the annual coupon is $69? A) $999.55 B) $1,002.01 C) $1,007.45 D) $1,012.13

B

A bond has a par value of $1,000, a time to maturity of 10 years, and a coupon rate of 8% with interest paid annually. If the current market price is $750, what is the capital gain yield of this bond over the next year? A) .72% B) 1.85% C) 2.58% D) 3.42%

B

A bond pays a semiannual coupon, and the last coupon was paid 61 days ago. If the annual coupon payment is $75, what is the accrued interest? (Assume 182 days in the 6-month period.) A) $13.21 B) $12.57 C) $15.44 D) $16.32

B

A bond was purchased at a premium and is now selling at a discount because of a change in market interest rates. If the bond pays a 4% annual coupon, what is the likely impact on the holding-period return if an investor decides to sell now? A) increased B) decreased C) stayed the same D) The answer cannot be determined from the information given.

B

A call option on Brocklehurst Corp. has an exercise price of $30. The current stock price of Brocklehurst Corp. is $32. The call option is ________. A) at the money B) in the money C) out of the money D) knocked in

B

A common stock pays an annual dividend per share of $1.80. The risk-free rate is 5%, and the risk premium for this stock is 4%. If the annual dividend is expected to remain at $1.80 per share, what is the value of the stock? A) $17.78 B) $20 C) $40 D) none of these options

B

A company with an expected earnings growth rate which is greater than that of the typical company in the same industry most likely has ________. A) a dividend yield which is greater than that of the typical company B) a dividend yield which is less than that of the typical company C) less risk than the typical company D) less sensitivity to market trends than the typical company

B

A convertible bond has a par value of $1,000, but its current market price is $950. The current price of the issuing company's stock is $19, and the conversion ratio is 40 shares. The bond's conversion premium is ________. A) $50 B) $190 C) $200 D) $240

B

A convertible bond has a par value of $1,000, but its current market price is $975. The current price of the issuing company's stock is $26, and the conversion ratio is 34 shares. The bond's market conversion value is ________. A) $1,000 B) $884 C) $933 D) $980

B

A convertible bond is deep in the money. This means the bond price will closely track the ________. A) straight debt value of the bond B) conversion value of the bond C) straight debt value of the bond minus the conversion value D) straight debt value of the bond plus the conversion value

B

A corporation in a 34% tax bracket invests in the preferred stock of another company and earns a 6% pretax rate of return. An individual investor in a 15% tax bracket invests in the same preferred stock and earns the same pretax return. The after-tax return to the corporation is ________, and the after-tax return to the individual investor is ________. A) 3.96%; 5.1% B) 5.39%; 5.1% C) 6%; 6% D) 3.96%; 6%

B

A coupon bond that pays interest annually has a par value of $1,000, matures in 5 years, and has a yield to maturity of 12%. If the coupon rate is 9%, the intrinsic value of the bond today will be ________. A) $856.04 B) $891.86 C) $926.47 D) $1,000

B

A coupon bond that pays interest of $60 annually has a par value of $1,000, matures in 5 years, and is selling today at a $75.25 discount from par value. The current yield on this bond is ________. A) 6% B) 6.49% C) 6.73% D) 7%

B

A coupon bond that pays interest semiannually has a par value of $1,000, matures in 8 years, and has a yield to maturity of 6%. If the coupon rate is 7%, the intrinsic value of the bond today will be ________. A) $1,000 B) $1,062.81 C) $1,081.82 D) $1,100.03

B

A covered call strategy benefits from what environment? A) Falling interest rates B) Price stability C) Price volatility D) Unexpected events

B

A firm cuts its dividend payout ratio. As a result, you know that the firm's ________. A) return on assets will increase B) earnings retention ratio will increase C) earnings growth rate will fall D) stock price will fall

B

A firm has PVGO of 0 and a market capitalization rate of 12%. What is the firm's P/E ratio? A) 12 B) 8.33 C) 10.25 D) 18.55

B

A firm has an earnings retention ratio of 40%. The stock has a market capitalization rate of 15% and an ROE of 18%. What is the stock's P/E ratio? A) 12.82 B) 7.69 C) 8.33 D) 9.46

B

A firm is expected to produce earnings next year of $3 per share. It plans to reinvest 25% of its earnings at 20%. If the cost of equity is 11%, what should be the value of the stock? A) $27.27 B) $37.50 C) $66.67 D) $70

B

A firm's earnings per share increased from $10 to $12, its dividends increased from $4 to $4.40, and its share price increased from $80 to $100. Given this information, it follows that ________. A) the stock experienced a drop in its P/E ratio B) the company had a decrease in its dividend payout ratio C) both earnings and share price increased by 20% D) the required rate of return increased

B

A measure of the riskiness of an asset held in isolation is ________. A) beta B) standard deviation C) covariance D) alpha

B

A mutual fund that attempts to hold quantities of shares in proportion to their representation in the market is called a __________ fund. A. stock B. index C. hedge D. money market

B

A portfolio is composed of two stocks, A and B. Stock A has a standard deviation of return of 35%, while stock B has a standard deviation of return of 15%. The correlation coefficient between the returns on A and B is .45. Stock A comprises 40% of the portfolio, while stock B comprises 60% of the portfolio. The standard deviation of the return on this portfolio is ________. A) 23% B) 19.76% C) 18.45% D) 17.67%

B

A portfolio of stocks fluctuates when the Treasury yields change. Since this risk cannot be eliminated through diversification, it is called ________. A) firm-specific risk B) systematic risk C) unique risk D) none of the options

B

A project has a 50% chance of doubling your investment in 1 year and a 50% chance of losing half your money. What is the expected return on this investment project? A) 0% B) 25% C) 50% D) 75%

B

A put option on Dr. Pepper Snapple Group, Inc., has an exercise price of $45. The current stock price is $41. The put option is ________. A) at the money B) in the money C) out of the money D) knocked out

B

A security with normally distributed returns has an annual expected return of 18% and standard deviation of 23%. The probability of getting a return between -28% and 64% in any one year is ________. A) 68.26% B) 95.44% C) 99.74% D) 100%

B

A stock has an intrinsic value of $15 and an actual stock price of $13.50. You know that this stock ________. A) has a Tobin's q value < 1 B) will generate a positive alpha C) has an expected return less than its required return D) has a beta > 1

B

ART has come out with a new and improved product. As a result, the firm projects an ROE of 25%, and it will maintain a plowback ratio of 0.20. Its earnings this year will be $3 per share. Investors expect a 12% rate of return on the stock. At what P/E ratio would you expect ART to sell? A) 8.33 B) 11.43 C) 14.29 D) 15.25

B

ART has come out with a new and improved product. As a result, the firm projects an ROE of 25%, and it will maintain a plowback ratio of 0.20. Its earnings this year will be $3 per share. Investors expect a 12% rate of return on the stock. At what price would you expect ART to sell? A) $25 B) $34.29 C) $42.86 D) $45.67

B

ART has come out with a new and improved product. As a result, the firm projects an ROE of 25%, and it will maintain a plowback ratio of 0.20. Its earnings this year will be $3 per share. Investors expect a 12% rate of return on the stock. What is the present value of growth opportunities for ART? A) $8.57 B) $9.29 C) $14.29 D) $16.29

B

According to SEC Rule 415 regarding shelf registration, firms can gradually sell securities to the public for ________ following initial registration. A) 1 year B) 2 years C) 3 years D) 4 years

B

According to capital asset pricing theory, the key determinant of portfolio returns is ________. A) the degree of diversification B) the systematic risk of the portfolio C) the firm-specific risk of the portfolio D) economic factors

B

According to the capital asset pricing model, a fairly priced security will plot ________. A) above the security market line B) along the security market line C) below the security market line D) at no relation to the security market line

B

According to the capital asset pricing model, in equilibrium ________. A) all securities' returns must lie below the capital market line B) all securities' returns must lie on the security market line C) the slope of the security market line must be less than the market risk premium D) any security with a beta of 1 must have an excess return of zero

B

According to the semistrong form of the efficient markets hypothesis, ____________. A. stock prices do not rapidly adjust to new information B. future changes in stock prices cannot be predicted from any information that is publicly available C. corporate insiders should have no better investment performance than other investors even if allowed to trade freely D. arbitrage between futures and cash markets should not produce extraordinary profits

B

All major stock markets today are effectively ________. A) specialist trading systems B) electronic trading systems C) continuous auction markets D) direct search markets

B

An Asian call option gives its holder the right to ________. A) buy the underlying asset at the exercise price on or before the expiration date B) buy the underlying asset at a price determined by the average stock price during some specified portion of the option's life C) sell the underlying asset at the exercise price on or before the expiration date D) sell the underlying asset at a price determined by the average stock price during some specified portion of the option's life

B

An investment earns 10% the first year, earns 15% the second year, and loses 12% the third year. The total compound return over the 3 years was ________. A) 41.68% B) 11.32% C) 3.64% D) 13%

B

An investor buys a T-bill at a bank discount quote of 4.80 with 150 days to maturity for $9800. The bill has a face value of $10,000. The investor's bond equivalent yield on this investment is ________. A) 4.8% B) 4.97% C) 5.47% D) 5.74%

B

An investor can design a risky portfolio based on two stocks, A and B. Stock A has an expected return of 18% and a standard deviation of return of 20%. Stock B has an expected return of 14% and a standard deviation of return of 5%. The correlation coefficient between the returns of A and B is .50. The risk-free rate of return is 10%. The standard deviation of return on the optimal risky portfolio is ________. A) 0% B) 5% C) 7% D) 20%

B

An investor can design a risky portfolio based on two stocks, A and B. Stock A has an expected return of 21% and a standard deviation of return of 39%. Stock B has an expected return of 14% and a standard deviation of return of 20%. The correlation coefficient between the returns of A and B is .4. The risk-free rate of return is 5%. The expected return on the optimal risky portfolio is approximately ________. (Hint: Find weights first.) A) 14% B) 16% C) 18% D) 19%

B

An investor can design a risky portfolio based on two stocks, A and B. The standard deviation of return on stock A is 20%, while the standard deviation on stock B is 15%. The correlation coefficient between the returns on A and B is 0%. The rate of return for stocks A and B is 20% and 10% respectively. The expected return on the minimum-variance portfolio is approximately ________. A) 10% B) 13.6% C) 15% D) 19.41%

B

An investor purchases one municipal bond and one corporate bond that pay rates of return of 5% and 6.4%, respectively. If the investor is in the 15% tax bracket, his after-tax rates of return on the municipal and corporate bonds would be, respectively, ________. A) 5% and 6.4% B) 5% and 5.44% C) 4.25% and 6.4% D) 5.75% and 5.44%

B

An investor should do which of the following for stocks with negative alphas? A) go long B) sell short C) hold D) do nothing

B

An order to buy or sell a security at the current price is a ________. A) limit order B) market order C) stop-loss order D) stop-buy order

B

Arbitrage is ________. A) an example of the law of one price B) the creation of riskless profits made possible by relative mispricing among securities C) a common opportunity in modern markets D) an example of a risky trading strategy based on market forecasting

B

Assuming all other factors remain unchanged, ________ would increase a firm's price-earnings ratio. A) an increase in the dividend payout ratio B) a reduction in investor risk aversion C) an expected increase in the level of inflation D) an increase in the yield on Treasury bills

B

Based on the outcomes in the following table, choose which of the statements below is (are) correct? Scenario Security A Security B Security C Recession Return > E(r) Return = E(r) Return < E(r) Normal Return = E(r) Return = E(r) Return = E(r) Boom Return < E(r) Return = E(r) Return > E(r) I. The covariance of security A and security B is zero. II. The correlation coefficient between securities A and C is negative. III. The correlation coefficient between securities B and C is positive. A) I only B) I and II only C) II and III only D) I, II, and III

B

Beta is a measure of ________. A) total risk B) relative systematic risk C) relative nonsystematic risk D) relative business risk

B

Bonds rated ________ or better by Standard & Poor's are considered investment grade. A) AA B) BBB C) BB D) CCC

B

Both investors and gamblers take on risk. The difference between an investor and a gambler is that an investor ________. A) is normally risk neutral B) requires a risk premium to take on the risk C) knows he or she will not lose money D) knows the outcomes at the beginning of the holding period

B

Cache Creek Manufacturing Company is expected to pay a dividend of $3.36 in the upcoming year. Dividends are expected to grow at 8% per year. The risk-free rate of return is 4%, and the expected return on the market portfolio is 14%. Investors use the CAPM to compute the market capitalization rate and use the constant-growth DDM to determine the value of the stock. The stock's current price is $84. Using the constant-growth DDM, the market capitalization rate is ________. A) 9% B) 12% C) 14% D) 18%

B

Cache Creek Manufacturing Company is expected to pay a dividend of $4.20 in the upcoming year. Dividends are expected to grow at the rate of 8% per year. The risk-free rate of return is 4%, and the expected return on the market portfolio is 14%. Investors use the CAPM to compute the market capitalization rate on the stock and use the constant-growth DDM to determine the intrinsic value of the stock. The stock is trading in the market today at $84. Using the constant-growth DDM and the CAPM, the beta of the stock is ________. A) 1.4 B) 0.9 C) 0.8 D) 0.5

B

Compensation of money managers is ________ based on alpha or other appropriate risk-adjusted measures. A) never B) rarely C) almost always D) always

B

Consider the expectations theory of the term structure of interest rates. If the yield curve is downward-sloping, this indicates that investors expect short-term interest rates to ________ in the future. A) increase B) decrease C) not change D) change in an unpredictable manner

B

Consider the following two investment alternatives: First, a risky portfolio that pays a 15% rate of return with a probability of 40% or a 5% rate of return with a probability of 60%. Second, a Treasury bill that pays 6%. The risk premium on the risky investment is ________. A) 1% B) 3% C) 6% D) 9%

B

Consider the following two investment alternatives: First, a risky portfolio that pays a 20% rate of return with a probability of 60% or a 5% rate of return with a probability of 40%. Second, a Treasury bill that pays 6%. If you invest $50,000 in the risky portfolio, your expected profit after one year would be ________. A) $3,000 B) $7,000 C) $7,500 D) $10,000

B

Consider the liquidity preference theory of the term structure of interest rates. On average, one would expect investors to require ________. A) a higher yield on short-term bonds than on long-term bonds B) a higher yield on long-term bonds than on short-term bonds C) the same yield on both short-term bonds and long-term bonds D) none of these options (The liquidity preference theory cannot be used to make any of the other statements.)

B

Consider the single factor APT. Portfolio A has a beta of 1.3 and an expected return of 21%. Portfolio B has a beta of .7 and an expected return of 17%. The risk-free rate of return is 8%. If you wanted to take advantage of an arbitrage opportunity, you should take a short position in portfolio ________ and a long position in portfolio ________. A) A; A B) A; B C) B; A D) B; B

B

Decreasing the number of stocks in a portfolio from 50 to 10 would likely ________. A) increase the systematic risk of the portfolio B) increase the unsystematic risk of the portfolio C) increase the return of the portfolio D) decrease the variation in returns the investor faces in any one year

B

Diversification is most effective when security returns are ________. A) high B) negatively correlated C) positively correlated D) uncorrelated

B

During the 1926-2013 period the Sharpe ratio was greatest for which of the following asset classes? A) small U.S. stocks B) large U.S. stocks C) long-term U.S. Treasury bonds D) bond world portfolio return in U.S. dollars

B

Even if the markets are efficient, professional portfolio management is still important because it provides investors with:I. Low-cost diversificationII. A portfolio with a specified risk levelIII. Better risk-adjusted returns than an index A. I only B. I and II only C. II and III only D. I, II, and III

B

Everything else equal, the ________ the maturity of a bond and the ________ the coupon, the greater the sensitivity of the bond's price to interest rate changes. A) longer; higher B) longer; lower C) shorter; higher D) shorter; lower

B

Exercise prices for listed stock options usually occur in increments of ________ and bracket the current stock price. A) $1 B) $5 C) $20 D) $25

B

Explicit costs of a stock IPO tend to be around ________ of the funds raised. A) 1% B) 7% C) 15% D) 25%

B

Fama and French claim that after controlling for firm size and the ratio of the firm's book value to market value, beta is: I. Highly significant in predicting future stock returns II. Relatively useless in predicting future stock returns III. A good predictor of the firm's specific risk A) I only B) II only C) I and III only D) I, II, and III

B

From 1926 to 2013 the world stock portfolio offered ________ return and ________ volatility than the portfolio of large U.S. stocks. A) lower; higher B) lower; lower C) higher; lower D) higher; higher

B

Gagliardi Way Corporation has an expected ROE of 15%. If it pays out 30% of its earnings as dividends, its dividend growth rate will be ________. A) 4.5% B) 10.5% C) 15% D) 30%

B

Generally speaking, as a firm progresses through the industry life cycle, you would expect the PVGO to ________ as a percentage of share price. A) increase B) decrease C) stay the same D) No typical pattern can be expected.

B

Grott and Perrin, Inc., has expected earnings of $3 per share for next year. The firm's ROE is 20%, and its earnings retention ratio is 70%. If the firm's market capitalization rate is 15%, what is the present value of its growth opportunities? A) $20 B) $70 C) $90 D) $115

B

If the beta of the market index is 1 and the standard deviation of the market index increases from 12% to 18%, what is the new beta of the market index? A) .8 B) 1 C) 1.2 D) 1.5

B

If the gross profit is positive and the net profit is negative, you will ________. A) let the option expire with no action B) not exercise the option C) exercise the option D) sell the option for less than the gross profit

B

If the quote for a Treasury bond is listed in the newspaper as 99.25 bid, 99.26 ask, the actual price at which you can sell this bond given a $10,000 par value is ________. A) $9,828.12 B) $9,925 C) $9,934.37 D) $9,955.43

B

If you anticipate a dramatic decline in stock prices, which naked strategy will make you the most profit? A) long call B) long put C) short call D) short put

B

If you are promised a nominal return of 12% on a 1-year investment, and you expect the rate of inflation to be 3%, what real rate do you expect to earn? A) 5.48% B) 8.74% C) 9% D) 12%

B

If you believe in the __________ form of the EMH, you believe that stock prices reflect all relevant information, including information that is available only to insiders. A. semistrong B. strong C. weak D. perfect

B

In 2013, NYSE Euronext was acquired by ________. A) DOT B) ICE C) BATS D) It was not acquired.

B

In ________ markets, participants post bid and ask prices at which they are willing to trade, but orders are not automatically executed by computer. ________ execute trades for people other than themselves, and in ________ markets a computer matches orders with an existing limit order book and executes the trades automatically. A) electronic; Dealers; brokers B) dealer; Brokers; electronic C) direct search; Brokers; electronic D) brokered; Dealers; direct search

B

In a ________ underwriting arrangement, the underwriter assumes the full risk that shares may not be sold to the public at the stipulated offering price. A) best-efforts B) firm-commitment C) private placement D) none of these options

B

In a study conducted by Jagannathan and Wang, it was found that the performance of beta in explaining security returns could be considerably enhanced by: I. Including the unsystematic risk of a stock II. Including human capital in the market portfolio III. Allowing for changes in beta over time A) I and II only B) II and III only C) I and III only D) I, II, and III

B

In a world where the CAPM holds, which one of the following is not a true statement regarding the capital market line? A) The capital market line always has a positive slope. B) The capital market line is also called the security market line. C) The capital market line is the best-attainable capital allocation line. D) The capital market line is the line from the risk-free rate through the market portfolio.

B

In an efficient market and for an investor who believes in a passive approach to investing, what is the primary duty of a portfolio manager? A. Accounting for results B. Diversification C. Identifying undervalued stocks D. No need for a portfolio manager

B

In calculating the Dow Jones Industrial Average, the adjustment for a stock split occurs ________. A) automatically B) by adjusting the divisor C) by adjusting the numerator D) by adjusting the market value weights

B

In calculating the variance of a portfolio's returns, squaring the deviations from the mean results in: I. Preventing the sum of the deviations from always equaling zero II. Exaggerating the effects of large positive and negative deviations III. A number for which the unit is percentage of returns A) I only B) I and II only C) I and III only D) I, II, and III

B

In the context of the capital asset pricing model, the systematic measure of risk is captured by ________. A) unique risk B) beta C) the standard deviation of returns D) the variance of returns

B

Initial margin requirements on stocks are set by ________. A) the Federal Deposit Insurance Corporation B) the Federal Reserve C) the New York Stock Exchange D) the Securities and Exchange Commission

B

Investors will earn higher rates of returns on TIPS than on equivalent default-risk standard bonds if ________. A) inflation is lower than anticipated over the investment period B) inflation is higher than anticipated over the investment period C) the U.S. dollar increases in value against the euro D) the spread between commercial paper and Treasury securities remains low

B

Joe bought a stock at $57 per share. The price promptly fell to $55. Joe held on to the stock until it again reached $57, and then he sold it once he had eliminated his loss. If other investors do the same to establish a trading pattern, this would contradict _______. A. the strong-form EMH B. the weak-form EMH C. technical analysis D. the semistrong-form EMH

B

June call and put options on King Books Inc. are available with exercise prices of $30, $35, and $40. Among the different exercise prices, the call option with the ________ exercise price and the put option with the ________ exercise price will have the greatest value. A) $40; $30 B) $30; $40 C) $35; $35 D) $40; $40

B

Market risk is also called ________ and ________. A) systematic risk; diversifiable risk B) systematic risk; nondiversifiable risk C) unique risk; nondiversifiable risk D) unique risk; diversifiable risk

B

Most studies indicate that investors' risk aversion is in the range ________. A) 1-3 B) 1.5-4 C) 3-5.2 D) 4-6

B

New-economy companies generally have higher ________ than old-economy companies. A) book value per share B) P/E multiples C) profits D) asset values

B

Next year's earnings are estimated to be $6. The company plans to reinvest 33% of its earnings at 12%. If the cost of equity is 8%, what is the present value of growth opportunities? A) $6 B) $24.50 C) $44.44 D) $75

B

Ownership of a put option entitles the owner to the ________ to ________ a specific stock, on or before a specific date, at a specific price. A) right; buy B) right; sell C) obligation; buy D) obligation; sell

B

Preferred stock is like long-term debt in that ________. A) it gives the holder voting power regarding the firm's management B) it promises to pay to its holder a fixed stream of income each year C) the preferred dividend is a tax-deductible expense for the firm D) in the event of bankruptcy preferred stock has equal status with debt

B

Published data on past returns earned by mutual funds are required to be ________. A) dollar-weighted returns B) geometric returns C) excess returns D) index returns

B

Purchases of new issues of stock take place ________. A) at the desk of the Fed B) in the primary market C) in the secondary market D) in the money markets

B

Rational risk-averse investors will always prefer portfolios ________. A) located on the efficient frontier to those located on the capital market line B) located on the capital market line to those located on the efficient frontier C) at or near the minimum-variance point on the risky asset efficient frontier D) that are risk-free to all other asset choices

B

Research has identified two systematic factors that affect U.S. stock returns. The factors are growth in industrial production and changes in long-term interest rates. Industrial production growth is expected to be 3%, and long-term interest rates are expected to increase by 1%. You are analyzing a stock that has a beta of 1.2 on the industrial production factor and .5 on the interest rate factor. It currently has an expected return of 12%. However, if industrial production actually grows 5% and interest rates drop 2%, what is your best guess of the stock's return? A) 15.9% B) 12.9% C) 13.2% D) 12%

B

Security A has a higher standard deviation of returns than security B. We would expect that: I. Security A would have a risk premium equal to security B. II. The likely range of returns for security A in any given year would be higher than the likely range of returns for security B. III. The Sharpe ratio of A will be higher than the Sharpe ratio of B. A) I only B) II only C) II and III only D) I, II, and III

B

Security A has an expected rate of return of 12% and a beta of 1.1. The market expected rate of return is 8%, and the risk-free rate is 5%. The alpha of the stock is ________. A) -1.7% B) 3.7% C) 5.5% D) 8.7%

B

Security X has an expected rate of return of 13% and a beta of 1.15. The risk-free rate is 5%, and the market expected rate of return is 15%. According to the capital asset pricing model, security X is ________. A) fairly priced B) overpriced C) underpriced D) none of these answers

B

Several large banks manipulated the reported rates on which key money market rate? A) federal funds rate B) LIBOR C) bankers' acceptances D) brokers' calls rate

B

Some diversification benefits can be achieved by combining securities in a portfolio as long as the correlation between the securities is ________. A) 1 B) less than 1 C) between 0 and 1 D) less than or equal to 0

B

Specialists try to maintain a narrow bid-ask spread because: I. If the spread is too large, they will not participate in as many trades, losing commission income. II. The exchange requires specialists to maintain price continuity. III. Specialists are nonprofit entities designed to facilitate market transactions rather than make a profit. A) I only B) I and II only C) II and III only D) I, II, and III

B

Stock market analysts have tended to be ___________ in their recommendations to investors. A. slightly overly optimistic B. overwhelmingly optimistic C. slightly overly pessimistic D. overwhelmingly pessimistic

B

Stock prices that are stable over time _______. A. indicate that prices are useful indicators of true economic value B. indicate that the market is not incorporating new information into current stock prices C. ensure that an economy allocates its resources efficiently D. indicates that returns follow a random-walk process

B

Stockholders of Dogs R Us Pet Supply expect a 12% rate of return on their stock. Management has consistently been generating an ROE of 15% over the last 5 years but now believes that ROE will be 12% for the next 5 years. Given this, the firm's optimal dividend payout ratio is now ________. A) 0% B) 100% C) between 0% and 50% D) between 50% and 100%

B

Suppose that a stock portfolio and a bond portfolio have a zero correlation. This means that ________. A) the returns on the stock and bond portfolios tend to move inversely B) the returns on the stock and bond portfolios tend to vary independently of each other C) the returns on the stock and bond portfolios tend to move together D) the covariance of the stock and bond portfolios will be positive

B

Suppose you find two bonds identical in all respects except that bond A is convertible to common stock and bond B is not. Bond A is priced at $1,245, and bond B is priced at $1,120. Bond A has a promised yield to maturity of 5.6%, and bond B has a promised yield to maturity of 6.7%. The stock of bond A is trading at $49.80 per share. Which of the following statements is (are) correct? I. The value of the conversion option for bond A is $125. II. The lower promised yield to maturity of bond A indicates that the bond is priced according to its straight debt value rather than its conversion value. III. If bond A can be converted into 25 shares of stock, the investor would break even at the current prices. A) II only B) I and III only C) III only D) I, II, and III

B

Suppose you pay $9,700 for a $10,000 par Treasury bill maturing in 3 months. What is the holding-period return for this investment? A) 3.01% B) 3.09% C) 12.42% D) 16.71%

B

T-bills are issued with initial maturities of: I. 4 weeks II. 16 weeks III. 26 weeks IV. 32 weeks A) I and II only B) I and III only C) I, II, and III only D) I, II, III, and IV

B

The EBIT of a firm is $300, the tax rate is 35%, the depreciation is $20, capital expenditures are $60, and the increase in net working capital is $30. What is the free cash flow to the firm? A) $85 B) $125 C) $185 D) $305

B

The Manhawkin Fund has an expected return of 16% and a standard deviation of 20%. The risk-free rate is 4%. What is the reward-to-volatility ratio for the Manhawkin Fund? A) .8 B) .6 C) 9 D) 1

B

The PEG ratio normalizes the P/E ratio by the A) tax rate. B) growth rate. C) market cap. D) book rate.

B

The U.K. stock index is the ________. A) DAX B) FTSE C) GSE D) TSE

B

The ________ is the covariance divided by the product of the standard deviations of the returns on each fund. A) covariance B) correlation coefficient C) standard deviation D) reward-to-variability ratio

B

The ________ is the most important dealer market in the United States, and the ________ is the most important auction market. A) NYSE; NASDAQ B) NASDAQ; NYSE C) CME; OTC D) AMEX; NYSE

B

The ________ measure of returns ignores compounding. A) geometric average B) arithmetic average C) IRR D) dollar-weighted

B

The ________ of a bond is computed as the ratio of the annual coupon payment to the market price. A) nominal yield B) current yield C) yield to maturity D) yield to call

B

The ________ price is the price at which a dealer is willing to sell a security. A) bid B) ask C) clearing D) settlement

B

The ________ requires full disclosure of relevant information relating to the issue of new securities. A) Insider Trading Act of 1931 B) Securities Act of 1933 C) Securities Exchange Act of 1934 D) Investment Company Act of 1940

B

The arbitrage pricing theory was developed by ________. A) Henry Markowitz B) Stephen Ross C) William Sharpe D) Eugene Fama

B

The arithmetic average of -11%, 15%, and 20% is ________. A) 15.67% B) 8% C) 11.22% D) 6.45%

B

The average depth of the limit order book is ________. A) lower for the large stocks in the S&P 500 Index than for the smaller stocks in the Russell 2000 Index B) higher for the large stocks in the S&P 500 Index than for the smaller stocks in the Russell 2000 Index C) about the same for both the large stocks in the S&P 500 Index and the smaller stocks in the Russell 2000 Index D) unrelated to the sizes of the stocks in the indexes

B

The bid price of a Treasury bill is ________. A) the price at which the dealer in Treasury bills is willing to sell the bill B) the price at which the dealer in Treasury bills is willing to buy the bill C) greater than the ask price of the Treasury bill expressed in dollar terms D) the price at which the investor can buy the Treasury bill

B

The brokers' call rate represents A) the rate the broker charges an investor on a margin account. B) the rate the broker pays its bank on borrowed funds. C) the return earned by the broker on a margin account. D) the return earned by the investor on a margin account.

B

The difference between the price at which a dealer is willing to buy and the price at which a dealer is willing to sell is called the ________. A) market spread B) bid-ask spread C) bid-ask gap D) market variation

B

The excess return is the ________. A) rate of return that can be earned with certainty B) rate of return in excess of the Treasury-bill rate C) rate of return to risk aversion D) index return

B

The expected return of a portfolio is 8.9%, and the risk-free rate is 3.5%. If the portfolio standard deviation is 12%, what is the reward-to-variability ratio of the portfolio? A) 0 B) .45 C) .74 D) 1.35

B

The expected return on the market is the risk-free rate plus the ________. A) diversified returns B) equilibrium risk premium C) historical market return D) unsystematic return

B

The free cash flow to the firm is $300 million in perpetuity, the cost of equity equals 14%, and the WACC is 10%. If the market value of the debt is $1 billion, what is the value of the equity using the free cash flow valuation approach? A) $1 billion B) $2 billion C) $3 billion D) $4 billion

B

The free cash flow to the firm is reported as $205 million. The interest expense to the firm is $22 million. If the tax rate is 35% and the net debt of the firm increased by $25 million, what is the approximate market value of the firm if the FCFE grows at 2% and the cost of equity is 11%? A) $2,168 billion B) $2,445 billion C) $2,565 billion D) $2,998 billion

B

The fully automated trade-execution system installed on the NYSE is called ________. A) FAX B) Direct + C) NASDAQ D) SUPERDOT

B

The holding period return on a stock is equal to ________. A) the capital gain yield over the period plus the inflation rate B) the capital gain yield over the period plus the dividend yield C) the current yield plus the dividend yield D) the dividend yield plus the risk premium

B

The market capitalization rate on the stock of Aberdeen Wholesale Company is 10%. Its expected ROE is 12%, and its expected EPS is $5. If the firm's plowback ratio is 50%, its P/E ratio will be ________. A) 8.33 B) 12.5 C) 19.23 D) 24.15

B

The market capitalization rate on the stock of Aberdeen Wholesale Company is 10%. Its expected ROE is 12%, and its expected EPS is $5. If the firm's plowback ratio is 60%, its P/E ratio will be ________. A) 7.14 B) 14.29 C) 16.67 D) 22.22

B

The possibility of arbitrage arises when ________. A) there is no consensus among investors regarding the future direction of the market, and thus trades are made arbitrarily B) mispricing among securities creates opportunities for riskless profits C) two identically risky securities carry the same expected returns D) investors do not diversify

B

The primary market where new security issues are offered to the public is a good example of ________. A) an auction market B) a brokered market C) a dealer market D) a direct search market

B

The process of polling potential investors regarding their interest in a forthcoming initial public offering (IPO) is called ________. A) interest building B) book building C) market analysis D) customer identification

B

The rate of return on ________ is known at the beginning of the holding period, while the rate of return on ________ is not known until the end of the holding period. A) risky assets; Treasury bills B) Treasury bills; risky assets C) excess returns; risky assets D) index assets; bonds

B

The risk that can be diversified away is ________. A) beta B) firm-specific risk C) market risk D) systematic risk

B

The risk-free rate is 4%. The expected market rate of return is 11%. If you expect stock X with a beta of .8 to offer a rate of return of 12%, then you should ________. A) buy stock X because it is overpriced B) buy stock X because it is underpriced C) sell short stock X because it is overpriced D) sell short stock X because it is underpriced

B

The semistrong form of the EMH states that ________ must be reflected in the current stock price. A. all security price and volume data B. all publicly available information C. all information, including inside information D. all costless information

B

The standard deviation of return on investment A is 10%, while the standard deviation of return on investment B is 4%. If the correlation coefficient between the returns on A and B is -.50, the covariance of returns on A and B is ________. A) -.0447 B) -.0020 C) .0020 D) .0447

B

The term excess return refers to ________. A) returns earned illegally by means of insider trading B) the difference between the rate of return earned and the risk-free rate C) the difference between the rate of return earned on a particular security and the rate of return earned on other securities of equivalent risk D) the portion of the return on a security that represents tax liability and therefore cannot be reinvested

B

The term inside quotes refers to ________. A) the difference between the lowest bid price and the highest ask price in the limit order book. B) the difference between the highest bid price and the lowest ask price in the limit order book. C) the difference between the lowest bid price and the lowest ask price in the limit order book. D) the difference between the highest bid price and the highest ask price in the limit order book.

B

The term latency refers to ________. A) the lag between when an order is placed on the NYSE and when it is executed. B) the amount of time it takes to accept, process, and deliver a trading order. C) the time it takes to implement new rules and procedures for stock exchanges and computer trading systems. D) the lag between when an order is executed and when the investor takes possession of the securities.

B

The two-factor model on a stock provides a risk premium for exposure to market risk of 9%, a risk premium for exposure to interest rate risk of (-1.3%), and a risk-free rate of 3.5%. The beta for exposure to market risk is 1, and the beta for exposure to interest rate risk is also 1. What is the expected return on the stock? A) 8.7% B) 11.2% C) 13.8% D) 15.2%

B

The variance of the return on the market portfolio is .04 and the expected return on the market portfolio is 20%. If the risk-free rate of return is 10%, the market degree of risk aversion, A, is ________. A) .5 B) 2.5 C) 3.5 D) 5

B

The writer of a put option ________. A) agrees to sell shares at a set price if the option holder desires B) agrees to buy shares at a set price if the option holder desires C) has the right to buy shares at a set price D) has the right to sell shares at a set price

B

Three stocks have share prices of $12, $75, and $30 with total market values of $400 million, $350 million, and $150 million, respectively. If you were to construct a price-weighted index of the three stocks, what would be the index value? A) 300 B) 39 C) 43 D) 30

B

To earn a high rating from the bond rating agencies, a company would want to have: I. A low times-interest-earned ratio II. A low debt-to-equity ratio III. A high quick ratio A) I only B) II and III only C) I and III only D) I, II, and III

B

Todd Mountain Development Corporation is expected to pay a dividend of $2.50 in the upcoming year. Dividends are expected to grow at the rate of 8% per year. The risk-free rate of return is 5%, and the expected return on the market portfolio is 12%. The stock of Todd Mountain Development Corporation has a beta of 0.75. Using the CAPM, the return you should require on the stock is ________. A) 7.25% B) 10.25% C) 14.75% D) 21%

B

Transactions that do not involve the original issue of securities take place in ________. A) primary markets B) secondary markets C) over-the-counter markets D) institutional markets

B

Transportation stocks currently provide an expected rate of return of 15%. TTT, a large transportation company, will pay a year-end dividend of $3 per share. If the stock is selling at $60 per share, what must be the market's expectation of the constant-growth rate of TTT dividends? A) 5% B) 10% C) 20% D) none of these options

B

Treasury bills are financial instruments issued by ________ to raise funds. A) commercial banks B) the federal government C) large corporations D) state and city governments

B

Underwriting is one of the services provided by ________. A) the SEC B) investment bankers C) publicly traded companies D) FDIC

B

Using the index model, the alpha of a stock is 3%, the beta is 1.1, and the market return is 10%. What is the residual given an actual return of 15%? A) .0% B) 1% C) 2% D) 3%

B

Westsyde Tool Company is expected to pay a dividend of $2 in the upcoming year. The risk-free rate of return is 6%, and the expected return on the market portfolio is 12%. Analysts expect the price of Westsyde Tool Company shares to be $29 a year from now. The beta of Westsyde Tool Company's stock is 1.2. Using a one-period valuation model, the intrinsic value of Westsyde Tool Company stock today is ________. A) $24.29 B) $27.39 C) $31.13 D) $34.52

B

Weyerhaeuser Incorporated has a balance sheet that lists $70 million in assets, $45 million in liabilities, and $25 million in common shareholders' equity. It has 1 million common shares outstanding. The replacement cost of its assets is $85 million. Its share price in the market is $49. Its book value per share is ________. A) $16.67 B) $25 C) $37.50 D) $40.83

B

What is the expected return on a stock with a beta of .8, given a risk-free rate of 3.5% and an expected market return of 15.5%? A) 3.8% B) 13.1% C) 15.6% D) 19.1%

B

What is the standard deviation of a portfolio of two stocks given the following data: Stock A has a standard deviation of 30%. Stock B has a standard deviation of 18%. The portfolio contains 60% of stock A, and the correlation coefficient between the two stocks is -1. A) 0% B) 10.8% C) 18% D) 24%

B

What strategy could be considered insurance for an investment in a portfolio of stocks? A) covered call B) protective put C) short put D) straddle

B

What would you expect to have happened to the spread between yields on commercial paper and Treasury bills immediately after September 11, 2001? A) no change, as both yields will remain the same B) increase, as the spread usually increases in response to a crisis C) decrease, as the spread usually decreases in response to a crisis D) no change, as both yields will move in the same direction

B

When Google's share price reached $475 per share, Google had a P/E ratio of about 68 and an estimated market capitalization rate of 11.5%. Google pays no dividends. Approximately what percentage of Google's stock price was represented by PVGO? A) 92% B) 87% C) 77% D) 64%

B

When computing the bank discount yield, you would use ________ days in the year. A) 260 B) 360 C) 365 D) 366

B

When issued, most convertible bonds are issued ________. A) deep in the money B) deep out of the money C) slightly out of the money D) slightly in the money

B

When the market risk premium rises, stock prices will ________. A. rise B. fall C. recover D. have excess volatility

B

Which of the following are assumptions of the simple CAPM model? I. Individual trades of investors do not affect a stock's price. II. All investors plan for one identical holding period. III. All investors analyze securities in the same way and share the same economic view of the world. IV. All investors have the same level of risk aversion. A) I, II, and IV only B) I, II, and III only C) II, III, and IV only D) I, II, III, and IV

B

Which of the following arguments supporting passive investment strategies is (are) correct? I. Active trading strategies may not guarantee higher returns but guarantee higher costs. II. Passive investors can free-ride on the activity of knowledge investors whose trades force prices to reflect currently available information. III. Passive investors are guaranteed to earn higher rates of return than active investors over sufficiently long time horizons. A) I only B) I and II only C) II and III only D) I, II, and III

B

Which of the following bonds would most likely sell at the lowest yield? A) a callable debenture B) a puttable mortgage bond C) a callable mortgage bond D) a puttable debenture

B

Which of the following correlation coefficients will produce the most diversification benefits? A) -.6 B) -.9 C) 0 D) .4

B

Which of the following is most like a short-term collateralized loan? A) certificate of deposit B) repurchase agreement C) bankers' acceptance D) commercial paper

B

Which of the following is not a characteristic of common stock ownership? A) residual claimant B) unlimited liability C) voting rights D) right to any dividend paid by the corporation.

B

Which of the following is not a method employed by fundamental analysts? A. Analyzing the Fed's next interest rate move B. Relative strength analysis C. Earnings forecasting D. Estimating the economic growth rate

B

Which of the following is not a true statement regarding municipal bonds? A) A municipal bond is a debt obligation issued by state or local governments. B) A municipal bond is a debt obligation issued by the federal government. C) The interest income from a municipal bond is exempt from federal income taxation. D) The interest income from a municipal bond is exempt from state and local taxation in the issuing state.

B

Which of the following is used to back international sales of goods and services? A) certificate of deposit B) bankers' acceptance C) eurodollar deposits D) commercial paper

B

Which of the following rates represents a bond's annual interest payment per dollar of par value? A) holding period return B) coupon rate C) IRR D) YTM

B

Which of the following statistics cannot be negative? A) covariance B) variance C) E(r) D) correlation coefficient

B

Which one of the following is a false statement regarding NYSE specialists? A) On a stock exchange most buy or sell orders are executed via an electronic system rather than through specialists. B) Specialists cannot trade for their own accounts. C) Specialists maintain limit order books, which contain the outstanding unexecuted limit orders. D) Specialists stand ready to trade at narrower bid-ask spreads in cases where the spread has become too wide.

B

Which one of the following is a true statement regarding the Dow Jones Industrial Average? A) It is a value-weighted average of 30 large industrial stocks. B) It is a price-weighted average of 30 large industrial stocks. C) It is a price-weighted average of 100 large stocks traded on the New York Stock Exchange. D) It is a value-weighted average of all stocks traded on the New York Stock Exchange.

B

Which one of the following is a true statement? A) Dividends on preferred stocks are tax-deductible to individual investors but not to corporate investors. B) Common dividends cannot be paid if preferred dividends are in arrears on cumulative preferred stock. C) Preferred stockholders have voting power. D) Investors can sue managers for nonpayment of preferred dividends.

B

Which one of the following statements is correct? A) invoice price = flat price - accrued interest B) invoice price = flat price + accrued interest C) flat price = invoice price + accrued interest D) invoice price = settlement price - accrued interest

B

You are considering acquiring a common share of Sahali Shopping Center Corporation that you would like to hold for 1 year. You expect to receive both $1.25 in dividends and $35 from the sale of the share at the end of the year. The maximum price you would pay for a share today is ________ if you wanted to earn a 12% return. A) $31.25 B) $32.37 C) $38.47 D) $41.32

B

You are considering investing $1,000 in a complete portfolio. The complete portfolio is composed of Treasury bills that pay 5% and a risky portfolio, P, constructed with two risky securities, X and Y. The optimal weights of X and Y in P are 60% and 40%, respectively. X has an expected rate of return of 14%, and Y has an expected rate of return of 10%. If you decide to hold 25% of your complete portfolio in the risky portfolio and 75% in the Treasury bills, then the dollar values of your positions in X and Y, respectively, would be ________ and ________. A) $300; $450 B) $150; $100 C) $100; $150 D) $450; $300

B

You are considering investing $1,000 in a complete portfolio. The complete portfolio is composed of Treasury bills that pay 5% and a risky portfolio, P, constructed with two risky securities, X and Y. The optimal weights of X and Y in P are 60% and 40%, respectively. X has an expected rate of return of 14%, and Y has an expected rate of return of 10%. The dollar values of your positions in X, Y, and Treasury bills would be ________, ________, and ________, respectively, if you decide to hold a complete portfolio that has an expected return of 8%. A) $162; $595; $243 B) $243; $162; $595 C) $595; $162; $243 D) $595; $243; $162

B

You are considering purchasing the Zions Bank $4.50 preferred stock. If you require a 4% return on this investment, what should you be willing to pay for this stock? A) $11.25 B) $112.50 C) $4.50 D) $45.00

B

You are convinced that a stock's price will move by at least 15% over the next 3 months. You are not sure which way the price will move, but you believe that the results of a patent hearing are definitely going to have a major effect on the stock price. You are somewhat more bullish than bearish however. Which one of the following options strategies best fits this scenario? A) buy a strip. B) buy a strap. C) buy a straddle. D) write a straddle.

B

You buy a call option and a put option on General Electric. Both the call option and the put option have the same exercise price and expiration date. This strategy is called a ________. A) time spread B) long straddle C) short straddle D) money spread

B

You have an EAR of 9%. The equivalent APR with continuous compounding is ________. A) 8.47% B) 8.62% C) 8.88% D) 9.42%

B

You have calculated the historical dollar-weighted return, annual geometric average return, and annual arithmetic average return. You always reinvest your dividends and interest earned on the portfolio. Which method provides the best measure of the actual average historical performance of the investments you have chosen? A) dollar-weighted return B) geometric average return C) arithmetic average return D) index return

B

You have the following rates of return for a risky portfolio for several recent years. Assume that the stock pays no dividends. Year Beginning of Year Price # of Shares Bought or Sold 2014 $ 50.00 100 bought 2015 $ 55.00 50 bought 2016 $ 51.00 75 sold 2017 $ 54.00 75 sold What is the dollar-weighted return over the entire time period? A) 2.87% B) .74% C) 2.6% D) 2.21%

B

You have the following rates of return for a risky portfolio for several recent years: 2013 35.23% 2014 18.67% 2015 −9.87% 2016 23.45% If you invested $1,000 at the beginning of 2013, your investment at the end of 2016 would be worth ________. A) $2,176.60 B) $1,785.56 C) $1,645.53 D) $1,247.87

B

You invest all of your money in 1-year T-bills. Which of the following statements is (are) correct? I. Your nominal return on the T-bills is riskless. II. Your real return on the T-bills is riskless. III. Your nominal Sharpe ratio is zero. A) I only B) I and III only C) II only D) I, II, and III

B

You own a stock portfolio worth $50,000. You are worried that stock prices may take a dip before you are ready to sell, so you are considering purchasing either at-the-money or out-of-the-money puts. If you decide to purchase the out-of-the-money puts, your maximum loss is ________ than if you buy at-the-money puts and your maximum gain is ________. A) greater; lower B) greater; greater C) lower; greater D) lower; lower

B

You purchase one MBI July 120 call contract (equaling 100 shares) for a premium of $5. You hold the option until the expiration date, when MBI stock sells for $123 per share. You will realize a ________ on the investment. A) $200 profit B) $200 loss C) $300 profit D) $300 loss

B

You purchase one MBI July 120 put contract (equaling 100 shares) for a premium of $3. You hold the option until the expiration date, when MBI stock sells for $123 per share. You will realize a ________ on the investment. A) $300 profit B) $300 loss C) $500 loss D) $200 profit

B

You purchased 200 shares of ABC common stock on margin at $50 per share. Assume the initial margin is 50% and the maintenance margin is 30%. You will get a margin call if the stock drops below ________. (Assume the stock pays no dividends, and ignore interest on the margin loan.) A) $26.55 B) $35.71 C) $28.95 D) $30.77

B

You sell one Huge-Packing August 50 call contract and sell one Huge-Packing August 50 put contract. The call premium is $1.25 and the put premium is $4.50. Your strategy will pay off only if the stock price is ________ in August. A) either lower than $44.25 or higher than $55.75 B) between $44.25 and $55.75 C) higher than $55.75 D) lower than $44.25

B

You sell short 300 shares of Microsoft that are currently selling at $30 per share. You post the 50% margin required on the short sale. If you earn no interest on the funds in your margin account, what will be your rate of return after 1 year if Microsoft is selling at $27? (Ignore any dividends.) A) 10% B) 20% C) 6.67% D) 15%

B

________ is the amount of money per common share that could be realized by breaking up the firm, selling its assets, repaying its debt, and distributing the remainder to shareholders. A) Book value per share B) Liquidation value per share C) Market value per share D) Tobin's q

B

________ is the most risky transaction to undertake in the stock-index option markets if the stock market is expected to fall substantially after the transaction is completed. A) Writing an uncovered call option B) Writing an uncovered put option C) Buying a call option D) Buying a put option

B

________ would not be included in the EAFE index. A) Australia B) Canada C) France D) Japan

B

$1,000 par value zero-coupon bonds (ignore liquidity premiums) Bond A/B/C/D/E Years to Maturity 1/2/3/4/5 Yield to Maturity 6.00%/7.50%/7.99%/8.49%/10.70% The expected 2-year interest rate 3 years from now should be ________. A) 9.55% B) 11.74% C) 14.89% D) 13.73%

C

$1,000 par value zero-coupon bonds (ignore liquidity premiums) Bond A/B/C/D/E Years to Maturity 1/2/3/4/5 Yield to Maturity 6.00%/7.50%/7.99%/8.49%/10.70% The expected 1-year interest rate 1 year from now should be about ________. A) 6% B) 7.5 % C) 9.02% D) 10.08%

C

A "bet" option is also called a ________ option. A) barrier B) lookback C) digital D) foreign exchange

C

A Japanese firm issued and sold a pound-denominated bond in the United Kingdom. A U.S. firm issued bonds denominated in dollars but sold the bonds in Japan. Which one of the following statements is correct? A) Both bonds are examples of Eurobonds. B) The Japanese bond is a Eurobond, and the U.S. bond is termed a foreign bond. C) The U.S. bond is a Eurobond, and the Japanese bond is termed a foreign bond. D) Neither bond is a Eurobond.

C

A ________ bond gives the bondholder the right to cash in the bond before maturity at a specific price after a specific date. A) callable B) coupon C) puttable D) Treasury

C

A benchmark index has three stocks priced at $23, $43, and $56. The number of outstanding shares for each is 350,000 shares, 405,000 shares, and 553,000 shares, respectively. If the market value weighted index was 970 yesterday and the prices changed to $23, $41, and $58 today, what is the new index value? A) 960 B) 970 C) 975 D) 985

C

A benchmark market value index is comprised of three stocks. Yesterday the three stocks were priced at $12, $20, and $60. The number of outstanding shares for each is 600,000 shares, 500,000 shares, and 200,000 shares, respectively. If the stock prices changed to $16, $18, and $62 today respectively, what is the 1-day rate of return on the index? A) 5.78% B) 4.35% C) 6.16% D) 7.42%

C

A bond issued by the state of Alabama is priced to yield 6.25%. If you are in the 28% tax bracket, this bond would provide you with an equivalent taxable yield of ________. A) 4.5% B) 7.25% C) 8.68% D) none of these options

C

A coupon bond that pays interest of $60 annually has a par value of $1,000, matures in 5 years, and is selling today at an $84.52 discount from par value. The yield to maturity on this bond is ________. A) 6% B) 7.23% C) 8.12% D) 9.45%

C

A coupon bond that pays semiannual interest is reported in the Wall Street Journal as having an ask price of 117% of its $1,000 par value. If the last interest payment was made 2 months ago and the coupon rate is 6%, the invoice price of the bond will be ________. A) $1,140 B) $1,170 C) $1,180 D) $1,200

C

A firm is planning on paying its first dividend of $2 three years from today. After that, dividends are expected to grow at 6% per year indefinitely. The stock's required return is 14%. What is the intrinsic value of a share today? A) $25 B) $16.87 C) $19.24 D) $20.99

C

A firm reports EBIT of $100 million. The income statement shows depreciation of $20 million. If the tax rate is 35% and total capital expenditures and increases in working capital total $10 million, what is the free cash flow to the firm? A) $57 B) $65 C) $75 D) $95

C

A futures call option provides its holder with the right to ________. A) purchase a particular stock at some time in the future at a specified price B) purchase a futures contract for the delivery of options on a particular stock C) purchase a futures contract at a specified price for a specified period of time D) deliver a futures contract and receive a specified price at a specific date in the future

C

A mortgage bond is ________. A) secured by other securities held by the firm B) secured by equipment owned by the firm C) secured by property owned by the firm D) unsecured

C

A portfolio with a 25% standard deviation generated a return of 15% last year when T-bills were paying 4.5%. This portfolio had a Sharpe ratio of ________. A) .22 B) .60 C) .42 D) .25

C

A stock is trading at $50. You believe there is a 60% chance the price of the stock will increase by 10% over the next 3 months. You believe there is a 30% chance the stock will drop by 5%, and you think there is only a 10% chance of a major drop in price of 20%. At-the-money 3-month puts are available at a cost of $650 per contract. What is the expected dollar profit for a writer of a naked put at the end of 3 months? A) $300 B) $200 C) $475 D) $0

C

A stock quote indicates a stock price of $60 and a dividend yield of 3%. The latest quarterly dividend received by stock investors must have been ________ per share. A) $0.55 B) $1.80 C) $0.45 D) $1.25

C

A tax free municipal bond provides a yield of 2.34%. What is the equivalent taxable yield on the bond given a 28% tax bracket? A) 2.34% B) 2.68% C) 3.25% D) 4.92%

C

A tax free municipal bond provides a yield of 3.2%. What is the equivalent taxable yield on the bond given a 35% tax bracket? A) 3.2% B) 3.68% C) 4.92% D) 5%

C

ART has come out with a new and improved product. As a result, the firm projects an ROE of 25%, and it will maintain a plowback ratio of 0.20. Its earnings this year will be $3 per share. Investors expect a 12% rate of return on the stock. What price do you expect ART shares to sell for in 4 years? A) $53.96 B) $44.95 C) $41.68 D) $39.76

C

According to the CAPM, the risk premium an investor expects to receive on any stock or portfolio is ________. A) directly related to the risk aversion of the particular investor B) inversely related to the risk aversion of the particular investor C) directly related to the beta of the stock D) inversely related to the alpha of the stock

C

According to the CAPM, which of the following is not a true statement regarding the market portfolio. A) All securities in the market portfolio are held in proportion to their market values. B) It includes all risky assets in the world, including human capital. C) It is always the minimum-variance portfolio on the efficient frontier. D) It lies on the efficient frontier.

C

According to the capital asset pricing model, a security with a ________. A) negative alpha is considered a good buy B) positive alpha is considered overpriced C) positive alpha is considered underpriced D) zero alpha is considered a good buy

C

Advantages of exchange-traded options over OTC options include all but which one of the following? A) ease and low cost of trading B) anonymity of participants C) contracts that are tailored to meet the needs of market participants D) no concerns about counterparty credit risk

C

An American put option gives its holder the right to ________. A) buy the underlying asset at the exercise price on or before the expiration date B) buy the underlying asset at the exercise price only at the expiration date C) sell the underlying asset at the exercise price on or before the expiration date D) sell the underlying asset at the exercise price only at the expiration date

C

An implication of the efficient market hypothesis is that __________. A. high-beta stocks are consistently overpriced B. low-beta stocks are consistently overpriced C. nonzero alphas will quickly disappear D. growth stocks are better buys than value stocks

C

An important characteristic of market equilibrium is ________. A) the presence of many opportunities for creating zero-investment portfolios B) all investors exhibit the same degree of risk aversion C) the absence of arbitrage opportunities D) the lack of liquidity in the market

C

An investor buys $16,000 worth of a stock priced at $20 per share using 60% initial margin. The broker charges 8% on the margin loan and requires a 35% maintenance margin. The stock pays a $.50-per-share dividend in 1 year, and then the stock is sold at $23 per share. What was the investor's rate of return? A) 17.5% B) 19.67% C) 23.83% D) 25.75%

C

An investor buys $8,000 worth of a stock priced at $40 per share using 50% initial margin. The broker charges 6% on the margin loan and requires a 30% maintenance margin. In 1 year the investor has interest payable and gets a margin call. At the time of the margin call the stock's price must have been less than ________. A) $20 B) $29.77 C) $30.29 D) $32.45

C

An investor can design a risky portfolio based on two stocks, A and B. Stock A has an expected return of 21% and a standard deviation of return of 39%. Stock B has an expected return of 14% and a standard deviation of return of 20%. The correlation coefficient between the returns of A and B is .4. The risk-free rate of return is 5%. The standard deviation of returns on the optimal risky portfolio is ________. A) 25.5% B) 22.3% C) 21.4% D) 20.7%

C

An investor can design a risky portfolio based on two stocks, A and B. The standard deviation of return on stock A is 20%, while the standard deviation on stock B is 15%. The correlation coefficient between the returns on A and B is 0%. The rate of return for stocks A and B is 20% and 10% respectively. The standard deviation of return on the minimum-variance portfolio is ________. A) 0% B) 6% C) 12% D) 17%

C

An investor can design a risky portfolio based on two stocks, A and B. The standard deviation of return on stock A is 24%, while the standard deviation on stock B is 14%. The correlation coefficient between the returns on A and B is .35. The expected return on stock A is 25%, while on stock B it is 11%. The proportion of the minimum-variance portfolio that would be invested in stock B is approximately ________. A) 45% B) 67% C) 85% D) 92%

C

An investor invests 70% of her wealth in a risky asset with an expected rate of return of 15% and a variance of 5%, and she puts 30% in a Treasury bill that pays 5%. Her portfolio's expected rate of return and standard deviation are ________ and ________ respectively. A) 10%; 6.7% B) 12%; 22.4% C) 12%; 15.7% D) 10%; 35%

C

An investor purchases a long call at a price of $2.50. The strike price at expiration is $35. If the current stock price is $35.10, what is the break-even point for the investor? A) $32.50 B) $35 C) $37.50 D) $37.60

C

An investor's degree of risk aversion will determine his or her ________. A) optimal risky portfolio B) risk-free rate C) optimal mix of the risk-free asset and risky asset D) capital allocation line

C

An option with a payoff that depends on the average price of the underlying asset during at least some portion of the life of the option is called ________ option. A) an American B) a European C) an Asian D) an Australian

C

An underpriced stock provides an expected return that is ________ the required return based on the capital asset pricing model (CAPM). A) less than B) equal to C) greater than D) greater than or equal to

C

Assume you purchased 500 shares of XYZ common stock on margin at $40 per share from your broker. If the initial margin is 60%, the amount you borrowed from the broker is ________. A) $20,000 B) $12,000 C) $8,000 D) $15,000

C

At contract maturity the value of a put option is ________, where X equals the option's strike price and ST is the stock price at contract expiration. A) max (0, ST − X) B) min (0, ST − X) C) max (0, X − ST) D) min (0, X − ST)

C

Bonds with coupon rates that fall when the general level of interest rates rise are called ________. A) asset-backed bonds B) convertible bonds C) inverse floaters D) index bonds

C

Brevik Builders has an expected ROE of 25%. Its dividend growth rate will be ________ if it follows a policy of paying 30% of earnings in the form of dividends. A) 5% B) 15% C) 17.5% D) 45%

C

Buyers of listed options ________ required to post margins, and writers of naked listed options ________ required to post margins. A) are; are not B) are; are C) are not; are D) are not; are not

C

Cash Cow, Inc. earned $750,000,000 last year. If it retains 40% of its earnings, and has 100 million shares outstanding, what was its dividend last year? A) $3.00 B) $3.75 C) $4.50 D) $7.50

C

Commercial paper is a short-term security issued by ________ to raise funds. A) the Federal Reserve B) the New York Stock Exchange C) large well-known companies D) all of these options

C

Consider an investment opportunity set formed with two securities that are perfectly negatively correlated. The global minimum-variance portfolio has a standard deviation that is always ________. A) equal to the sum of the securities' standard deviations B) equal to -1 C) equal to 0 D) greater than 0

C

Consider the CAPM. The expected return on the market is 18%. The expected return on a stock with a beta of 1.2 is 20%. What is the risk-free rate? A) 2% B) 6% C) 8% D) 12%

C

Consider the capital asset pricing model. The market degree of risk aversion, A, is 3. The risk premium is 2.25%. If the risk-free rate of return is 4%, the expected return on the market portfolio is ________. A) 6.75% B) 9% C) 10.75% D) 12%

C

Consider the following $1,000 par value zero-coupon bonds: Bond A/B/C/D/E Years to Maturity 1/2/3/4/5 Yield to Maturity 6.00%/7.00%/8.32%/8.49%/10.70% The expected 1-year interest rate 3 years from now should be ________. A) 7% B) 8% C) 9% D) 10%

C

Consider the following $1,000 par value zero-coupon bonds: Bond A/B/C/D/E Years to Maturity 1/2/3/4/5 Yield to Maturity 6.00%/7.50%/8.00%/8.50%/10.25% The expected 1-year interest rate 2 years from now should be ________. A) 7% B) 8% C) 9% D) 10%

C

Consider the multifactor APT with two factors. Portfolio A has a beta of .5 on factor 1 and a beta of 1.25 on factor 2. The risk premiums on the factor 1 and 2 portfolios are 1% and 7%, respectively. The risk-free rate of return is 7%. The expected return on portfolio A is ________ if no arbitrage opportunities exist. A) 13.5% B) 15% C) 16.25% D) 23%

C

Consider the one-factor APT. The standard deviation of return on a well-diversified portfolio is 20%. The standard deviation on the factor portfolio is 12%. The beta of the well-diversified portfolio is approximately ________. A) .60 B) 1 C) 1.67 D) 3.20

C

Consider the single factor APT. Portfolio A has a beta of .2 and an expected return of 13%. Portfolio B has a beta of .4 and an expected return of 15%. The risk-free rate of return is 10%. If you wanted to take advantage of an arbitrage opportunity, you should take a short position in portfolio ________ and a long position in portfolio ________. A) A; A B) A; B C) B; A D) B; B

C

Consider two perfectly negatively correlated risky securities, A and B. Security A has an expected rate of return of 16% and a standard deviation of return of 20%. B has an expected rate of return of 10% and a standard deviation of return of 30%. The weight of security B in the minimum-variance portfolio is ________. A) 10% B) 20% C) 40% D) 60%

C

Consider two stocks, A and B. Stock A has an expected return of 10% and a beta of 1.2. Stock B has an expected return of 14% and a beta of 1.8. The expected market rate of return is 9% and the risk-free rate is 5%. Security ________ would be considered the better buy because ________. A) A; it offers an expected excess return of .2% B) A; it offers an expected excess return of 2.2% C) B; it offers an expected excess return of 1.8% D) B; it offers an expected return of 2.4%

C

Diversification can reduce or eliminate ________ risk. A) all B) systematic C) nonsystematic D) only an insignificant

C

During the 1926-2013 period which one of the following asset classes provided the lowest real return? A) Small U.S. stocks B) Large U.S. stocks C) Long-term U.S. Treasury bonds D) Equity world portfolio in U.S. dollars

C

During the 1986-2013 period, the Sharpe ratio was lowest for which of the following asset classes? A) small U.S. stocks B) large U.S. stocks C) long-term U.S. Treasury bonds D) equity world portfolio in U.S. dollars

C

Each listed stock option contract gives the holder the right to buy or sell ________ shares of stock. A) 1 B) 10 C) 100 D) 1,000

C

Evidence supporting semistrong-form market efficiency suggests that investors should _________________________. A. rely on technical analysis to select securities B. rely on fundamental analysis to select securities C. use a passive trading strategy such as purchasing an index fund or an ETF D. select securities by throwing darts at the financial pages of the newspaper

C

Firm A has a stock price of $35, and 60% of the value of the stock is in the form of PVGO. Firm B also has a stock price of $35, but only 20% of the value of stock B is in the form of PVGO. We know that: I. Stock A will give us a higher return than Stock B. II. An investment in stock A is probably riskier than an investment in stock B. III. Stock A has higher forecast earnings growth than stock B. A) I only B) I and II only C) II and III only D) I, II, and III

C

Firm A is high-risk, and Firm B is low-risk. Everything else equal, which firm would you expect to have a higher P/E ratio? A) Firm A B) Firm B C) Both would have the same P/E if they were in the same industry. D) There is not necessarily any linkage between risk and P/E ratios.

C

Floating-rate bonds have a ________ that is adjusted with current market interest rates. A) maturity date B) coupon payment date C) coupon rate D) dividend yield

C

Historical returns have generally been ________ for stocks of small firms as (than) for stocks of large firms. A) the same B) lower C) higher D) none of these options (There is no evidence of a systematic relationship between returns on small-firm stocks and returns on large-firm stocks.)

C

Historically, small-firm stocks have earned higher returns than large-firm stocks. When viewed in the context of an efficient market, this suggests that ________. A) small firms are better run than large firms B) government subsidies available to small firms produce effects that are discernible in stock market statistics C) small firms are riskier than large firms D) small firms are not being accurately represented in the data

C

If a Treasury note has a bid price of $996.25, the quoted bid price in the Wall Street Journal would be ________. A) 99:5/8 B) 99:6/10 C) 99.6250 D) none of the options

C

If a firm has a free cash flow equal to $50 million and that cash flow is expected to grow at 3% forever, what is the total firm value given a WACC of 9.5%? A) $679.81 million B) $715.54 million C) $769.23 million D) $803.03 million

C

If a stock is correctly priced, then you know that ________. A) the dividend payout ratio is optimal B) the stock's required return is equal to the growth rate in earnings and dividends C) the sum of the stock's expected capital gain and dividend yield is equal to the stock's required rate of return D) the present value of growth opportunities is equal to the value of assets in place

C

If all investors become more risk averse, the SML will ________ and stock prices will ________. A) shift upward; rise B) shift downward; fall C) have the same intercept with a steeper slope; fall D) have the same intercept with a flatter slope; rise

C

If an investor places a ________ order, the stock will be sold if its price falls to the stipulated level. If an investor places a ________ order, the stock will be bought if its price rises above the stipulated level. A) buy stop; stop-loss B) market; limit C) stop-loss; buy stop D) limit; market

C

If the price of a $10,000 par Treasury bond is $10,237.50, the quote would be listed in the newspaper as ________. A) 102.237 B) 102.102 C) 102.375 D) 102.750

C

If the quote for a Treasury bond is listed in the newspaper as 98.2812 bid, 98.4062 ask, the actual price at which you can purchase this bond given a $10,000 par value is ________. A) $9,828.12 B) $9,809.38 C) $9,840.62 D) $9,813.42

C

If you are holding a premium bond, you must expect a ________ each year until maturity. If you are holding a discount bond, you must expect a ________ each year until maturity. (In each case assume that the yield to maturity remains stable over time.) A) capital gain; capital loss B) capital gain; capital gain C) capital loss; capital gain D) capital loss; capital loss

C

If you believe you have a 60% chance of doubling your money, a 30% chance of gaining 15%, and a 10% chance of losing your entire investment, what is your expected return? A) 5% B) 15% C) 54.5% D) 114.5%

C

If you combine a long stock position with selling an at-the-money call option, the resulting net payoff profile will resemble the payoff profile of a ________. A) long call B) short call C) short put D) long put

C

If you want to know the portfolio standard deviation for a three-stock portfolio, you will have to ________. A) calculate two covariances and one trivariance B) calculate only two covariances C) calculate three covariances D) average the variances of the individual stocks

C

If you want to measure the performance of your investment in a fund, including the timing of your purchases and redemptions, you should calculate the ________. A) geometric average return B) arithmetic average return C) dollar-weighted return D) index return

C

In an era of particularly low interest rates, which of the following bonds is most likely to be called? A) zero-coupon bonds B) coupon bonds selling at a discount C) coupon bonds selling at a premium D) floating-rate bonds

C

Inflation-indexed Treasury securities are commonly called ________. A) PIKs B) CARs C) TIPS D) STRIPS

C

Initial public offerings (IPOs) are usually ________ relative to the levels at which their prices stabilize after they begin trading in the secondary market. A) overpriced B) correctly priced C) underpriced D) mispriced, but without any particular bias

C

Insiders are able to profitably trade and earn abnormal returns prior to the announcement of positive news. This is a violation of which form of efficiency? A. Weak-form efficiency B. Semistrong-form efficiency C. Strong-form efficiency D. Technical analysis

C

Lifecycle Motorcycle Company is expected to pay a dividend in year 1 of $2, a dividend in year 2 of $3, and a dividend in year 3 of $4. After year 3, dividends are expected to grow at the rate of 7% per year. An appropriate required return for the stock is 12%. Using the multistage DDM, the stock should be worth ________ today. A) $63.80 B) $65.13 C) $67.95 D) $85.60

C

Many current and retired Enron Corp. employees had their 401k retirement accounts wiped out when Enron collapsed because ________. A) they had to pay huge fines for obstruction of justice B) their 401k accounts were held outside the company C) their 401k accounts were not well diversified D) none of these options

C

More than ________ of all trading is believed to be initiated by computer algorithms. A) 25% B) 40% C) 50% D) 75%

C

Most people would readily agree that the stock market is not _________. A. weak-form efficient B. semistrong-form efficient C. strong-form efficient D. efficient at all

C

Next year's earnings are estimated to be $5. The company plans to reinvest 20% of its earnings at 15%. If the cost of equity is 9%, what is the present value of growth opportunities? A) $9.09 B) $10.10 C) $11.11 D) $12.21

C

One extensive study found that about ________ of financial managers use CAPM to estimate cost of capital. A) one-third B) one-half C) three quarters D) ninety percent

C

One method of forecasting the risk premium is to use the ________. A) coefficient of variation of analysts' earnings forecasts B) variations in the risk-free rate over time C) average historical excess returns for the asset under consideration D) average abnormal return on the index portfolio

C

One-, two-, and three-year maturity, default-free, zero-coupon bonds have yields to maturity of 7%, 8%, and 9%, respectively. What is the implied 1-year forward rate 1 year from today? A) 2.07% B) 8.03% C) 9.01% D) 11.12%

C

P/E ratios tend to be ________ when inflation is ________. A) higher; higher B) lower; lower C) higher; lower D) they are unrelated

C

Proponents of the EMH typically advocate __________. A. a conservative investment strategy B. a liberal investment strategy C. a passive investment strategy D. an aggressive investment strategy

C

Rank the following from highest average historical return to lowest average historical return from 1926 to 2017. I. Small stocks II. Long-term bonds III. Large stocks IV. T-bills A) I, II, III, IV B) III, IV, II, I C) I, III, II, IV D) III, I, II, IV

C

Rank the following from highest average historical standard deviation to lowest average historical standard deviation from 1926 to 2017. I. Small stocks II. Long-term bonds III. Large stocks IV. T-bills A) I, II, III, IV B) III, IV, II, I C) I, III, II, IV D) III, I, II, IV

C

Regulation NMS: I. Supports the goal of integrating financial markets II. Requires the use of specialists to execute trades III. Requires that exchanges honor quotes of other exchanges when they can be executed automatically A) I only B) I and II only C) I and III only D) I, II, and III

C

Rose Hill Trading Company is expected to have EPS in the upcoming year of $6. The expected ROE is 18%. An appropriate required return on the stock is 14%. If the firm has a plowback ratio of 70%, its intrinsic value should be ________. A) $20.93 B) $69.77 C) $128.57 D) $150

C

Rose Hill Trading Company is expected to have EPS in the upcoming year of $8. The expected ROE is 18%. An appropriate required return on the stock is 14%. If the firm has a plowback ratio of 70%, its dividend in the upcoming year should be ________. A) $1.12 B) $1.44 C) $2.40 D) $5.60

C

SIPC ensures investors against failure of a brokerage firm up to a limit of ________. A) $100,000 B) $250,000 C) $500,000 D) $1,000,000

C

Sanders, Inc., paid a $4 dividend per share last year and is expected to continue to pay out 60% of its earnings as dividends for the foreseeable future. If the firm is expected to generate a 13% return on equity in the future, and if you require a 15% return on the stock, the value of the stock is ________. A) $26.67 B) $35.19 C) $42.94 D) $59.89

C

Sinking funds are commonly viewed as protecting the ________ of the bond. A) issuer B) underwriter C) holder D) dealer

C

Suppose you pay $9,400 for a $10,000 par Treasury bill maturing in 6 months. What is the effective annual rate of return for this investment? A) 6.38% B) 12.77% C) 13.17% D) 14.25%

C

Suppose you pay $9,800 for a $10,000 par Treasury bill maturing in 2 months. What is the annual percentage rate of return for this investment? A) 2.04% B) 12 % C) 12.24% D) 12.89%

C

TIPS offer investors inflation protection by ________ by the inflation rate each year. A) increasing only the coupon rate B) increasing only the par value C) increasing both the par value and the coupon payment D) increasing the promised yield to maturity

C

The CAL provided by combinations of 1-month T-bills and a broad index of common stocks is called the ________. A) SML B) CAPM C) CML D) total return line

C

The German stock market is measured by which market index? A) FTSE B) Dow Jones 30 C) DAX D) Nikkei

C

The Hydro Index is a price weighted stock index based on the 5 largest boat manufacturers in the nation. The stock prices for the five stocks are $10, $20, $80, $50 and $40. The price of the last stock was just split 2 for 1 and the stock price was halved from $40 to $20. What is the new divisor for a price weighted index? A) 5.00 B) 4.85 C) 4.50 D) 4.75

C

The SEC requires public U.S. companies to file registration statements and periodic reports electronically through A) Yahoo. B) Google. C) EDGAR. D) FINRA.

C

The ________ reward-to-variability ratio is found on the ________ capital market line. A) lowest; steepest B) highest; flattest C) highest; steepest D) lowest; flattest

C

The ________ was established to protect investors from losses if their brokerage firms fail. A) CFTC B) SEC C) SIPC D) AIMR

C

The bid-ask spread exists because of ________. A) market inefficiencies B) discontinuities in the markets C) the need for dealers to cover expenses and make a profit D) lack of trading in thin markets

C

The broadest information set is included in the _____. A. weak-form efficiency argument B. semistrong-form efficiency argument C. strong-form efficiency argument D. technical analysis trading method

C

The buyer of a new home is quoted a mortgage rate of .5% per month. What is the APR on the loan? A) .50% B) 5% C) 6% D) 6.5%

C

The capital asset pricing model was developed by ________. A) Kenneth French B) Stephen Ross C) William Sharpe D) Eugene Fama

C

The common stock of the Avalon Corporation has been trading in a narrow range around $40 per share for months, and you believe it is going to stay in that range for the next 3 months. The price of a 3-month put option with an exercise price of $40 is $3, and a call with the same expiration date and exercise price sells for $4. Suppose you write a strap and the stock price winds up to be $42 at contract expiration. What was your net profit on the strap? A) $200 B) $300 C) $700 D) $400

C

The common stock of the Avalon Corporation has been trading in a narrow range around $40 per share for months, and you believe it is going to stay in that range for the next 3 months. The price of a 3-month put option with an exercise price of $40 is $3, and a call with the same expiration date and exercise price sells for $4. What would be a simple options strategy using a put and a call to exploit your conviction about the stock price's future movement? A) sell a call B) purchase a put C) sell a straddle D) buy a straddle

C

The complete portfolio refers to the investment in ________. A) the risk-free asset B) the risky portfolio C) the risk-free asset and the risky portfolio combined D) the risky portfolio and the index

C

The constant-growth dividend discount model (DDM) can be used only when the ________. A) growth rate is less than or equal to the required return B) growth rate is greater than or equal to the required return C) growth rate is less than the required return D) growth rate is greater than the required return

C

The expected rate of return of a portfolio of risky securities is ________. A) the sum of the securities' covariance B) the sum of the securities' variance C) the weighted sum of the securities' expected returns D) the weighted sum of the securities' variance

C

The expected return on the market portfolio is 15%. The risk-free rate is 8%. The expected return on SDA Corp. common stock is 16%. The beta of SDA Corp. common stock is 1.25. Within the context of the capital asset pricing model, ________. A) SDA Corp. stock is underpriced B) SDA Corp. stock is fairly priced C) SDA Corp. stock's alpha is -.75% D) SDA Corp. stock alpha is .75%

C

The geometric average of -12%, 20%, and 25% is ________. A) 8.42% B) 11% C) 9.7% D) 18.88%

C

The graph of the relationship between expected return and beta in the CAPM context is called the ________. A) CML B) CAL C) SML D) SCL

C

The greatest value to an analyst from calculating a stock's intrinsic value is ________. A) how easy it is to come up with accurate model inputs B) the precision of the value estimate C) how the process forces analysts to understand the critical variables that have the greatest impact on value D) how all the different models typically yield identical value results

C

The holding-period return on a stock was 25%. Its ending price was $18, and its beginning price was $16. Its cash dividend must have been ________. A) $.25 B) $1 C) $2 D) $4

C

The holding-period return on a stock was 32%. Its beginning price was $25, and its cash dividend was $1.50. Its ending price must have been ________. A) $28.50 B) $33.20 C) $31.50 D) $29.75

C

The largest nongovernmental regulator of securities firms in the United States is ________. A) the CFA Institute B) the Public Company Accounting Oversight Board C) the Financial Industry Regulatory Authority D) the Board of Directors of NYSE Euronext

C

The margin requirement on a stock purchase is 25%. You fully use the margin allowed to purchase 100 shares of MSFT at $25. If the price drops to $22, what is your percentage loss? A) 9% B) 15% C) 48% D) 57%

C

The market share held by the "Other" category (which includes dark pools) constitutes roughly ________% of trading volume in NYSE-listed shares. A) 5% B) 10% C) 30% D) 50%

C

The most significant conceptual difference between the arbitrage pricing theory (APT) and the capital asset pricing model (CAPM) is that the CAPM ________. A) places less emphasis on market risk B) recognizes multiple unsystematic risk factors C) recognizes only one systematic risk factor D) recognizes multiple systematic risk factors

C

The optimal risky portfolio can be identified by finding: I. The minimum-variance point on the efficient frontier II. The maximum-return point on the efficient frontier and the minimum-variance point on the efficient frontier III. The tangency point of the capital market line and the efficient frontier IV. The line with the steepest slope that connects the risk-free rate to the efficient frontier A) I and II only B) II and III only C) III and IV only D) I and IV only

C

The over-the-counter securities market is a good example of ________. A) an auction market B) a brokered market C) a dealer market D) a direct search market

C

The price of a stock is $38 at the beginning of the year and $41 at the end of the year. If the stock paid a $2.50 dividend, what is the holding-period return for the year? A) 6.58% B) 8.86% C) 14.47% D) 18.66%

C

The price of a stock is $55 at the beginning of the year and $50 at the end of the year. If the stock paid a $3 dividend and inflation was 3%, what is the real holding-period return for the year? A) -3.64% B) -6.36% C) -6.44% D) -11.74%

C

The price quotations of Treasury bonds in the Wall Street Journal show a bid price of 104.5313 and an ask price of 104.5489. If you sell a Treasury bond, you expect to receive ________. A) $ 1,000.00 B) $ 1,045.00 C) $ 1,045.31 D) $ 1,045.48

C

The primary objective of fundamental analysis is to identify __________. A. well-run firms B. poorly run firms C. mispriced stocks D. high P/E stocks

C

The purchase of a futures contract gives the buyer ________. A) the right to buy an item at a specified price B) the right to sell an item at a specified price C) the obligation to buy an item at a specified price D) the obligation to sell an item at a specified price

C

The rate of interest on short-term loans among financial institutions is ________. A) bankers' acceptances B) brokers' calls C) federal funds rate D) LIBOR

C

The risk premium for exposure to aluminum commodity prices is 4%, and the firm has a beta relative to aluminum commodity prices of .6. The risk premium for exposure to GDP changes is 6%, and the firm has a beta relative to GDP of 1.2. If the risk-free rate is 4%, what is the expected return on this stock? A) 10% B) 11.5% C) 13.6% D) 14%

C

The risk-free rate and the expected market rate of return are 6% and 16%, respectively. According to the capital asset pricing model, the expected rate of return on security X with a beta of 1.2 is equal to ________. A) 12% B) 17% C) 18% D) 23%

C

The standard deviation of return on investment A is 10%, while the standard deviation of return on investment B is 5%. If the covariance of returns on A and B is .0030, the correlation coefficient between the returns on A and B is ________. A) .12 B) .36 C) .60 D) .77

C

The strong form of the EMH states that ________ must be reflected in the current stock price. A. all security price and volume data B. all publicly available information C. all information, including inside information D. all costless information

C

The term "residual claimant" refers to A) bond holders. B) option holders. C) equity/shareholders. D) suppliers.

C

The term "underwriting syndicate" describes ________. A) the issuing firm B) the lead underwriter C) the investment banks that participate in the underwriting D) the private investors that purchase the shares

C

The term random walk is used in investments to refer to ______________. A. stock price changes that are random but predictable B. stock prices that respond slowly to both old and new information C. stock price changes that are random and unpredictable D. stock prices changes that follow the pattern of past price changes

C

The value of Internet companies is based primarily on ________. A) current profits B) Tobin's q C) growth opportunities D) replacement cost

C

The value of a listed call option on a stock is lower when: I. The exercise price is higher. II. The contract approaches maturity. III. The stock decreases in value. IV. A stock split occurs. A) II, III, and IV only B) I, III, and IV only C) I, II, and III only D) I, II, III, and IV

C

Treasury bills are paying a 4% rate of return. A risk-averse investor with a risk aversion of A = 3 should invest entirely in a risky portfolio with a standard deviation of 24% only if the risky portfolio's expected return is at least ________. A) 8.67% B) 9.84% C) 21.28% D) 14.68%

C

Two assets have the following expected returns and standard deviations when the risk-free rate is 5%: Asset A E(rA) = 10% σA = 20% Asset B E(rB) = 15% σB = 27% An investor with a risk aversion of A = 3 would find that ________ on a risk-return basis. A) only asset A is acceptable B) only asset B is acceptable C) neither asset A nor asset B is acceptable D) both asset A and asset B are acceptable

C

Westsyde Tool Company is expected to pay a dividend of $1.50 in the upcoming year. The risk-free rate of return is 6%, and the expected return on the market portfolio is 14%. Analysts expect the price of Westsyde Tool Company shares to be $29 a year from now. The beta of Westsyde Tool Company's stock is 1.2. Using the CAPM, an appropriate required return on Westsyde Tool Company's stock is ________. A) 8% B) 10.8% C) 15.6% D) 16.8%

C

What is the geometric average return of the following quarterly returns: 3%, 5%, 4%, and 7%? A) 3.72% B) 4.23% C) 4.74% D) 4.90%

C

What is the geometric average return over 1 year if the quarterly returns are 8%, 9%, 5%, and 12%? A) 8% B) 8.33 % C) 8.47% D) 8.5 %

C

What is the lowest grade a bond can receive and still be considered investment grade? A) AAA B) A C) BBB D) BB

C

What is the most likely correlation coefficient between a stock-index mutual fund and the S&P 500? A) -1 B) 0 C) 1 D) .5

C

What would be the profit or loss per share of stock to an investor who bought an October expiration Apple call option with an exercise price of $130 if Apple closed on the expiration date at $120? Assume the option premium was $3.00. A) $0 B) $3.00 gain C) $3.00 loss D) $7.00 gain

C

Which country experienced the largest-ever sovereign default in 2012? A) Germany B) Ireland C) Greece D) Portugal

C

Which measure of downside risk predicts the worst loss that will be suffered with a given probability? A) standard deviation B) variance C) value at risk D) Sharpe ratio

C

Which of the following are true concerning short sales of exchange-listed stocks? I. Proceeds from the short sale must be kept on deposit with the broker. II. Short-sellers must post margin with their broker to cover potential losses on the position. III. The short-seller earns interest on any cash deposited with the broker that is used to meet the margin requirement. A) I only B) I and III only C) I and II only D) I, II, and III

C

Which of the following are true statements about T-bills? I. T-bills typically sell in denominations of $10,000. II. Income earned on T-bills is exempt from all federal taxes. III. Income earned on T-bills is exempt from state and local taxes. A) I only B) I and II only C) I and III only D) I, II, and III

C

Which of the following contradicts the proposition that the stock market is weakly efficient? A. Over 25% of mutual funds outperform the market on average. B. Insiders earn abnormal trading profits. C. Every January, the stock market earns above-normal returns. D. Applications of technical trading rules fail to earn abnormal returns.

C

Which of the following is not a method employed by followers of technical analysis? A. Charting B. Relative strength analysis C. Earnings forecasting D. Trading around support and resistance levels

C

Which of the following is not a nickname for an agency associated with the mortgage markets? A) Fannie Mae B) Freddie Mac C) Sallie Mae D) Ginnie Mae

C

Which of the following mortgage scenarios will benefit the homeowner the most? A) adjustable rate mortgage when interest rate increases. B) fixed rate mortgage when interest rates falls. C) fixed rate mortgage when interest rate rises. D) None of these options, as the banker's interest will always be protected.

C

Which of the following provides the best example of a systematic-risk event? A) A strike by union workers hurts a firm's quarterly earnings. B) Mad Cow disease in Montana hurts local ranchers and buyers of beef. C) The Federal Reserve increases interest rates 50 basis points. D) A senior executive at a firm embezzles $10 million and escapes to South America.

C

Which of the following stock price observations would appear to contradict the weak form of the efficient market hypothesis? A. The average rate of return is significantly greater than zero. B. The correlation between the market return one week and the return the following week is zero. C. You could have consistently made superior returns by buying stock after a 10% rise in price and selling after a 10% fall. D. You could have consistently made superior returns by forecasting future earnings performance with your new Crystal Ball forecast methodology.

C

Which of the following strategies makes a profit if the stock price stays stable? A) long call and short put B) long call and long put C) short call and short put D) short call and long put

C

Which of the following variables do Fama and French claim do a better job explaining stock returns than beta? I. Book-to-market ratio II. Unexpected change in industrial production III. Firm size A) I only B) I and II only C) I and III only D) I, II, and III

C

Which of the following would violate the efficient market hypothesis? A. Intel has consistently generated large profits for years. B. Prices for stocks before stock splits show, on average, consistently positive abnormal returns. C. Investors earn abnormal returns months after a firm announces surprise earnings. D. High-earnings growth stocks fail to generate higher returns for investors than do low earnings growth stocks.

C

Which one of the following is a common term for the market consensus value of the required return on a stock? A) dividend payout ratio B) intrinsic value C) market capitalization rate D) plowback ratio

C

Which one of the following is a true statement regarding corporate bonds? A) A corporate callable bond gives its holder the right to exchange it for a specified number of the company's common shares. B) A corporate debenture is a secured bond. C) A corporate convertible bond gives its holder the right to exchange it for a specified number of the company's common shares. D) Holders of corporate bonds have voting rights in the company.

C

Which one of the following provides the best example of securitization? A) convertible bond B) call option C) mortgage pass-through security D) preferred stock

C

Which one of the following statements about IPOs is not true? A) IPOs generally have been poor long-term investments. B) IPOs often provide very good initial returns to investors. C) IPOs generally provide superior long-term performance as compared to other stocks. D) Shares in IPOs are often primarily allocated to institutional investors.

C

Which one of the following stock return statistics fluctuates the most over time? A) covariance of returns B) variance of returns C) average return D) correlation coefficient

C

Which one of the following types of markets requires the greatest level of trading activity to be cost-effective? A) broker market B) dealer market C) continuous auction market D) direct search market

C

Which type of risk is most significant for bonds? A) maturity risk B) default risk C) interest rate risk D) reinvestment rate risk

C

Yields on municipal bonds are generally lower than yields on similar corporate bonds because of differences in ________. A) marketability B) risk C) taxation D) call protection

C

You are cautiously bullish on the common stock of the Wildwood Corporation over the next several months. The current price of the stock is $50 per share. You want to establish a bullish money spread to help limit the cost of your option position. You find the following option quotes: Wildwood Corp Underlying Stock price: $50.00 Expiration Strike Call Put June 45.00 8.50 2.00 June 50.00 4.50 3.00 June 55.00 2.00 7.50 If in June the stock price is $53, your net profit on the bull money spread (buy the 45 call and sell the 55 call) would be ________. A) $300 B) −$400 C) $150 D) $50

C

You are cautiously bullish on the common stock of the Wildwood Corporation over the next several months. The current price of the stock is $50 per share. You want to establish a bullish money spread to help limit the cost of your option position. You find the following option quotes: Wildwood Corp Underlying Stock price: $50.00 Expiration Strike Call Put June 45.00 8.50 2.00 June 50.00 4.50 3.00 June 55.00 2.00 7.50 To establish a bull money spread with calls, you would ________. A) buy the 55 call and sell the 45 call B) buy the 45 call and buy the 55 call C) buy the 45 call and sell the 55 call D) sell the 45 call and sell the 55 call

C

You are considering investing $1,000 in a complete portfolio. The complete portfolio is composed of Treasury bills that pay 5% and a risky portfolio, P, constructed with two risky securities, X and Y. The optimal weights of X and Y in P are 60% and 40% respectively. X has an expected rate of return of 14%, and Y has an expected rate of return of 10%. To form a complete portfolio with an expected rate of return of 8%, you should invest approximately ________ in the risky portfolio. This will mean you will also invest approximately ________ and ________ of your complete portfolio in security X and Y, respectively. A) 0%; 60%; 40% B) 25%; 45%; 30% C) 40%; 24%; 16% D) 50%; 30%; 20%

C

You believe that stock prices reflect all information that can be derived by examining market trading data such as the history of past stock prices, trading volume, or short interest, but you do not believe stock prices reflect all publicly available and inside information. You are a proponent of the ____________ form of the EMH. A. semistrong B. strong C. weak D. perfect

C

You buy a 10-year $1,000 par value 4% annual-payment coupon bond priced to yield 6%. You do not sell the bond at year-end. If you are in a 15% tax bracket, at year-end you will owe taxes on this investment equal to ________. A) $9.10 B) $4.25 C) $7.68 D) $5.20

C

You buy a 10-year $1,000 par value zero-coupon bond priced to yield 6%. You do not sell the bond. If you are in a 28% tax bracket, you will owe taxes on this investment after the first year equal to ________. A) $0 B) $4.27 C) $9.38 D) $33.51

C

You buy a TIPS at issue at par for $1,000. The bond has a 3% coupon. Inflation turns out to be 2%, 3%, and 4% over the next 3 years. The total annual coupon income you will receive in year 3 is ________. A) $30 B) $33 C) $32.78 D) $30.90

C

You buy one Huge-Packing August 50 call contract and one Huge-Packing August 50 put contract. The call premium is $1.25, and the put premium is $4.50. Your highest potential loss from this position is ________. A) $125 B) $450 C) $575 D) unlimited

C

You decide to purchase an equal number of shares of stocks of firms to create a portfolio. If you wanted to construct an index to track your portfolio performance, your best match for your portfolio would be to construct ________. A) a value-weighted index B) an equally weighted index C) a price-weighted index D) a bond price index

C

You find digital option quotes on jobless claims. You can buy a call option with a strike price of 300,000 jobless claims. This option pays $100 if actual claims exceed the strike price and pays zero otherwise. The option costs $68. A second digital call with a strike price of 305,000 jobless claims is available at a cost of $53. Suppose you buy the option with the 300,000 strike and sell the option with the 305,000 strike and jobless claims actually wind up at 303,000. Your net profit on the position is ________. A) −$15 B) $200 C) $85 D) $185

C

You have an APR of 7.5% with continuous compounding. The EAR is ________. A) 7.5% B) 7.65% C) 7.79 % D) 8.25%

C

You have calculated the historical dollar-weighted return, annual geometric average return, and annual arithmetic average return. If you desire to forecast performance for next year, the best forecast will be given by the ________. A) dollar-weighted return B) geometric average return C) arithmetic average return D) index return

C

You have the following rates of return for a risky portfolio for several recent years. Assume that the stock pays no dividends. Year Beginning of Year Price # of Shares Bought or Sold 2014 $ 50.00 100 bought 2015 $ 55.00 50 bought 2016 $ 51.00 75 sold 2017 $ 54.00 75 sold What is the geometric average return for the period? A) 2.87% B) .74% C) 2.6% D) 2.21%

C

You hold 5,000 shares of the 1 million outstanding shares of Wealthy Wranglers common stock. You've just learned that the company plans to issue more shares, so that 2 million shares will be outstanding. This is called ________. A) an advanced equity offering B) a weathered equity offering C) a seasoned equity offering D) a veteran equity offering

C

You hold a subordinated debenture in a firm. In the event of bankruptcy you will be paid off before which one of the following? A) mortgage bonds B) senior debentures C) preferred stock D) equipment obligation bonds

C

You invest $1,000 in a complete portfolio. The complete portfolio is composed of a risky asset with an expected rate of return of 16% and a standard deviation of 20% and a Treasury bill with a rate of return of 6%. The slope of the capital allocation line formed with the risky asset and the risk-free asset is approximately ________. A) 1.040 B) .80 C) .50 D) .25

C

You invest $1,000 in a complete portfolio. The complete portfolio is composed of a risky asset with an expected rate of return of 16% and a standard deviation of 20% and a Treasury bill with a rate of return of 6%. ________ of your complete portfolio should be invested in the risky portfolio if you want your complete portfolio to have a standard deviation of 9%. A) 100% B) 90% C) 45% D) 10%

C

You invest $600 in security A with a beta of 1.5 and $400 in security B with a beta of .90. The beta of this portfolio is ________. A) 1.14 B) 1.2 C) 1.26 D) 1.5

C

You invest in the stock of Valleyview Corp. and purchase a put option on Valleyview Corp. This strategy is called a ________. A) long straddle B) naked put C) protective put D) short stroll

C

You purchase one MBI March 120 put contract for a put premium of $10. The maximum profit that you could gain from this strategy is ________. A) $120 B) $1,000 C) $11,000 D) $12,000

C

You purchased 250 shares of common stock on margin for $25 per share. The initial margin is 65%, and the stock pays no dividend. Your rate of return would be ________ if you sell the stock at $32 per share. Ignore interest on margin. A) 35% B) 39% C) 43% D) 28%

C

You purchased a 5-year annual-interest coupon bond 1 year ago. Its coupon interest rate was 6%, and its par value was $1,000. At the time you purchased the bond, the yield to maturity was 4%. If you sold the bond after receiving the first interest payment and the bond's yield to maturity had changed to 3%, your annual total rate of return on holding the bond for that year would have been approximately ________. A) 5% B) 5.5% C) 7.6% D) 8.9%

C

You purchased a share of stock for $29. One year later you received $2.25 as dividend and sold the share for $28. Your holding-period return was ________. A) -3.57% B) -3.45% C) 4.31% D) 8.03%

C

You put half of your money in a stock portfolio that has an expected return of 14% and a standard deviation of 24%. You put the rest of your money in a risky bond portfolio that has an expected return of 6% and a standard deviation of 12%. The stock and bond portfolios have a correlation of .55. The standard deviation of the resulting portfolio will be ________. A) more than 18% but less than 24% B) equal to 18% C) more than 12% but less than 18% D) equal to 12%

C

You run a regression of a stock's returns versus a market index and find the following: Coefficients Lower 95% Upper 95% Intercept 0.789 -1.556 3.457 Slope 0.890 0.6541 1.465 Based on the data, you know that the stock ________. A) earned a positive alpha that is statistically significantly different from zero B) has a beta precisely equal to .890 C) has a beta that is likely to be anything between .6541 and 1.465 inclusive D) has no systematic risk

C

You sell one MBI July 90 call contract for a premium of $4 and two puts for a premium of $3 each. You hold the position until the expiration date, when MBI stock sells for $95 per share. You will realize a ________ on this strip. A) $300 profit B) $100 loss C) $500 profit D) $200 profit

C

You short-sell 200 shares of Tuckerton Trading Co., now selling for $50 per share. What is your maximum possible gain, ignoring transactions cost? A) $50 B) $150 C) $10,000 D) unlimited

C

You write a put option on a stock. The profit at contract maturity of the option position is ________, where X equals the option's strike price, ST is the stock price at contract expiration, and P0 is the original premium of the put option. A) max (P0, X − ST − P0) B) min (−P0, X − ST − P0) C) min (P0, ST − X + P0) D) max (0, ST − X − P0)

C

Your great aunt Zella invested $100 in 1925 in a portfolio of large U.S. stocks that earned a compound return of 10% annually.If she left that money to you, how much would be in the account 92 years later in 2017? A) $1,000 B) $9,900 C) $642,875.74 D) $5,843,325

C

________ are examples of synthetically created zero-coupon bonds. A) COLTS B) OPOSSMS C) STRIPS D) ARMs

C

________ bonds represent a novel way of obtaining insurance from capital markets against specified disasters. A) Asset-backed bonds B) TIPS C) Catastrophe D) Pay-in-kind

C

________ is considered to be an emerging market country. A) France B) Norway C) Brazil D) Canada

C

________ is not a money market instrument. A) A certificate of deposit B) A Treasury bill C) A Treasury bond D) Commercial paper

C

__________ is the return on a stock beyond what would be predicted from market movements alone. A. A normal return B. A subliminal return C. An abnormal return D. None of these options

C

A European put option gives its holder the right to ________. A) buy the underlying asset at the exercise price on or before the expiration date B) buy the underlying asset at the exercise price only at the expiration date C) sell the underlying asset at the exercise price on or before the expiration date D) sell the underlying asset at the exercise price only at the expiration date

D

A Treasury bond due in 1 year has a yield of 6.3%, while a Treasury bond due in 5 years has a yield of 8.8%. A bond due in 5 years issued by High Country Marketing Corp. has a yield of 9.6%, while a bond due in 1 year issued by High Country Marketing Corp. has a yield of 6.8%. The default risk premiums on the 1-year and 5-year bonds issued by High Country Marketing Corp. are, respectively, ________ and ________. A) .4%; .3% B) .4%; .5% C) .5%; .5% D) .5%; .8%

D

A corporate bond has a 10-year maturity and pays interest semiannually. The quoted coupon rate is 6%, and the bond is priced at par. The bond is callable in 3 years at 110% of par. What is the bond's yield to call? A) 6.72% B) 9.17% C) 4.49% D) 8.98%

D

A coupon bond that pays interest of 4% annually has a par value of $1,000, matures in 5 years, and is selling today at $785. The actual yield to maturity on this bond is ________. A) 7.24% B) 8.82% C) 9.12% D) 9.62%

D

A day trade with an average stock holding period of under 8 minutes might be most closely associated with which trading philosophy? A. EMH B. Fundamental analysis C. Strong-form market efficiency D. Technical analysis

D

A debenture is ________. A) secured by other securities held by the firm B) secured by equipment owned by the firm C) secured by property owned by the firm D) unsecured

D

A firm has a stock price of $55 per share and a P/E ratio of 75. If you buy the stock at this P/E and earnings fail to grow at all, how long should you expect it to take to just recover the cost of your investment? A) 27 years B) 37 years C) 55 years D) 75 years

D

A firm increases its dividend plowback ratio. All else equal, you know that ________. A) earnings growth will increase and the stock's P/E will increase B) earnings growth will decrease and the stock's P/E will increase C) earnings growth will increase and the stock's P/E will decrease D) earnings growth will increase and the stock's P/E may or may not increase

D

A firm that has large securities holdings and wishes to raise money for a short length of time may be able to find the cheapest financing from which of the following? A) reverse repurchase agreement B) bankers' acceptance C) commercial paper D) repurchase agreement

D

A loan for a new car costs the borrower .8% per month. What is the EAR? A) .80% B) 6.87% C) 9.6% D) 10.03%

D

A preferred share of Coquihalla Corporation will pay a dividend of $8 in the upcoming year and every year thereafter; that is, dividends are not expected to grow. You require a return of 7% on this stock. Using the constant-growth DDM to calculate the intrinsic value, a preferred share of Coquihalla Corporation is worth ________. A) $13.50 B) $45.50 C) $91 D) $114.29

D

A project has a 60% chance of doubling your investment in 1 year and a 40% chance of losing half your money. What is the standard deviation of this investment? A) 25% B) 50% C) 62% D) 73%

D

A quanto provides its holder with the right to ________. A) participate in the payoffs from a portfolio of gambling casino stocks B) exchange a fixed amount of a foreign currency for dollars at a specified exchange rate C) participate in the investment performance of a foreign security D) exchange the payoff from a foreign investment for dollars at a fixed exchange rate

D

A time spread may be executed by ________. A) selling an option with one exercise price and buying a similar one with a different exercise price B) buying two options that have the same expiration dates but different strike prices C) selling two options that have the same expiration dates but different strike prices D) selling an option with one expiration date and buying a similar option with a different expiration date

D

A typical bond price quote includes all but which one of the following? A) coupon B) closing bond price C) yield to maturity D) dividend yield

D

According to multiple studies by Ritter, initial public offerings tend to exhibit ________ performance initially and ________ performance over the long term. A) bad; good B) bad; bad C) good; good D) good; bad

D

According to the CAPM, investors are compensated for all but which of the following? A) expected inflation B) systematic risk C) time value of money D) residual risk

D

According to the CAPM, what is the expected market return given an expected return on a security of 15.8%, a stock beta of 1.2, and a risk-free interest rate of 5%? A) 5% B) 9% C) 13% D) 14%

D

According to the capital asset pricing model, fairly priced securities have ________. A) negative betas B) positive alphas C) positive betas D) zero alphas

D

Advantages of ECNs over traditional markets include all but which one of the following? A) lower transactions costs B) anonymity of the participants C) small amount of time needed to execute and order D) ability to handle very large orders

D

All but which one of the following indices is value weighted? A) NASDAQ Composite B) S&P 500 C) Wilshire 5000 D) DJIA

D

Among the important characteristics of market efficiency is (are) that: I. There are no arbitrage opportunities. II. Security prices react quickly to new information. III. Active trading strategies will not consistently outperform passive strategies. A. I only B. II only C. I and III only D. I, II, and III

D

An Asian put option gives its holder the right to ________. A) buy the underlying asset at the exercise price on or before the expiration date B) buy the underlying asset at a price determined by the average stock price during some specified portion of the option's life C) sell the underlying asset at the exercise price on or before the expiration date D) sell the underlying asset at a price determined by the average stock price during some specified portion of the option's life

D

An investor can design a risky portfolio based on two stocks, A and B. Stock A has an expected return of 21% and a standard deviation of return of 39%. Stock B has an expected return of 14% and a standard deviation of return of 20%. The correlation coefficient between the returns of A and B is .4. The risk-free rate of return is 5%. The proportion of the optimal risky portfolio that should be invested in stock B is approximately ________. A) 29% B) 44% C) 56% D) 71%

D

An investor in a 28% tax bracket is trying to decide whether to invest in a municipal bond or a corporate bond. She looks up municipal bond yields (rm) but wishes to calculate the taxable equivalent yield r. The formula she should use is given by ________. A) r = rm × (1 - 28%) B) r = rm / (1 - 72%) C) r = rm × (1 - 72%) D) r = rm / (1 - 28%)

D

An investor in a T-bill earns interest by ________. A) receiving interest payments every 90 days B) receiving dividend payments every 30 days C) converting the T-bill at maturity into a higher-valued T-note D) buying the bill at a discount from the face value to be received at maturity

D

Approximately how many securities does it take to diversify almost all of the unique risk from a portfolio? A) 2 B) 6 C) 8 D) 20

D

Barnegat Light sold 200,000 shares in an initial public offering. The underwriter's explicit fees were $90,000. The offering price for the shares was $35, but immediately upon issue, the share price jumped to $43. What is the best estimate of the total cost to Barnegat Light of the equity issue? A) $90,000 B) $1,290,000 C) $2,390,000 D) $1,690,000

D

Bill, Jim, and Shelly are all interested in buying the same stock that pays dividends. Bill plans on holding the stock for 1 year. Jim plans on holding the stock for 3 years. Shelly plans on holding the stock until she retires in 10 years. Which one of the following statements is correct? A) Bill will be willing to pay the most for the stock because he will get his money back in 1 year when he sells. B) Jim should be willing to pay three times as much for the stock as Bill will pay because his expected holding period is three times as long as Bill's. C) Shelly should be willing to pay the most for the stock because she will hold it the longest and hence will get the most dividends. D) All three should be willing to pay the same amount for the stock regardless of their holding period.

D

Building a zero-investment portfolio will always involve ________. A) an unknown mixture of short and long positions B) only short positions C) only long positions D) equal investments in a short and a long position

D

Caribou Gold Mining Corporation is expected to pay a dividend of $4 in the upcoming year. Dividends are expected to decline at the rate of 3% per year. The risk-free rate of return is 5%, and the expected return on the market portfolio is 13%. The stock of Caribou Gold Mining Corporation has a beta of 0.5. Using the CAPM, the return you should require on the stock is ________. A) 2% B) 5% C) 8% D) 9%

D

Consider the CAPM. The risk-free rate is 5%, and the expected return on the market is 15%. What is the beta on a stock with an expected return of 17%? A) .5 B) .7 C) 1 D) 1.2

D

Consider the CAPM. The risk-free rate is 6%, and the expected return on the market is 18%. What is the expected return on a stock with a beta of 1.3? A) 6% B) 15.6% C) 18% D) 21.6%

D

Consider the following limit order book of a specialist. The last trade in the stock occurred at a price of $40. If a market buy order for 100 shares comes in, at what price will it be filled? Limit Buy Price Orders Shares Limit Sell Orders Orders Shares $39.75 100 $40.25 100 $39.50 100 $40.50 100 A) $39.75 B) $40.25 C) $40.375 D) $40.25 or less

D

Consider two bonds, A and B. Both bonds presently are selling at their par value of $1,000. Each pays interest of $120 annually. Bond A will mature in 5 years, while bond B will mature in 6 years. If the yields to maturity on the two bonds change from 12% to 14%, ________. A) both bonds will increase in value but bond A will increase more than bond B B) both bonds will increase in value but bond B will increase more than bond A C) both bonds will decrease in value but bond A will decrease more than bond B D) both bonds will decrease in value but bond B will decrease more than bond A

D

Currently, the Dow Jones Industrial Average is computed by ________. A) adding the prices of 30 large "blue-chip" stocks and dividing by 30 B) calculating the total market value of the 30 firms in the index and dividing by 30 C) measuring the current total market value of the 30 stocks in the index relative to the total value on the previous day D) adding the prices of 30 large "blue-chip" stocks and dividing by a divisor adjusted for stock splits and large stock dividends

D

During the 1926-2013 period the geometric mean return on small-firm stocks was ________. A) 5.31% B) 5.56% C) 9.34% D) 11.82%

D

Empirical results estimated from historical data indicate that betas ________. A) are always close to zero B) are constant over time C) of all securities are always between zero and 1 D) seem to regress toward 1 over time

D

Everything else equal, which variable is negatively related to the intrinsic value of a company? A) D1 B) D0 C) g D) k

D

Exchange-traded stock options expire on the ________ of the expiration month. A) second Monday B) third Wednesday C) second Thursday D) third Friday

D

Firm-specific risk is also called ________ and ________. A) systematic risk; diversifiable risk B) systematic risk; nondiversifiable risk C) unique risk; nondiversifiable risk D) unique risk; diversifiable risk

D

Generally speaking, the higher a firm's ROA, the ________ the dividend payout ratio and the ________ the firm's growth rate of earnings. A) higher; lower B) higher; higher C) lower; lower D) lower; higher

D

If a firm increases its plowback ratio, this will probably result in ________ P/E ratio. A) a higher B) a lower C) an unchanged D) The answer cannot be determined from the information given.

D

If the simple CAPM is valid and all portfolios are priced correctly, which of the situations below is possible? Consider each situation independently, and assume the risk-free rate is 5%. A) Portfolio Expected Return Beta A 15% 1.2 Market 15% 1.0 B) Portfolio Expected Return Beta A 20% 12% Market 15% 20 C) Portfolio Expected Return Beta A 20% 1.2 Market 15% 1.0 D) Portfolio Expected Return Beta A 30% 2.5 Market 15% 1.0 A) Option A B) Option B C) Option C D) Option D

D

If you require a real growth in the purchasing power of your investment of 8%, and you expect the rate of inflation over the next year to be 3%, what is the lowest nominal return that you would be satisfied with? A) 3% B) 8% C) 11% D) 11.24%

D

In a simple CAPM world which of the following statements is (are) correct? I. All investors will choose to hold the market portfolio, which includes all risky assets in the world. II. Investors' complete portfolio will vary depending on their risk aversion. III. The return per unit of risk will be identical for all individual assets. IV. The market portfolio will be on the efficient frontier, and it will be the optimal risky portfolio. A) I, II, and III only B) II, III, and IV only C) I, III, and IV only D) I, II, III, and IV

D

In what industry are investors likely to use the dividend discount model and arrive at a price close to the observed market price? A) import/export trade B) software C) telecommunications D) utility

D

Investing in two assets with a correlation coefficient of -.5 will reduce what kind of risk? A) market risk B) nondiversifiable risk C) systematic risk D) unique risk

D

Investing in two assets with a correlation coefficient of 1 will reduce which kind of risk? A) market risk B) unique risk C) unsystematic risk D) none of these options (With a correlation of 1, no risk will be reduced.)

D

Investors require a risk premium as compensation for bearing ________. A) unsystematic risk B) alpha risk C) residual risk D) systematic risk

D

LIBOR is a key reference rate in the money markets. Many ________ of dollars of loans and derivative assets are tied to it. A) thousands B) millions C) billions D) trillions

D

Large well-known companies often issue their own short-term unsecured debt notes directly to the public, rather than borrowing from banks; their notes are called ________. A) certificates of deposit B) repurchase agreements C) bankers' acceptances D) commercial paper

D

Most tests of semistrong efficiency are _________. A. designed to test whether inside information can be used to generate abnormal returns B. based on technical trading rules C. unable to generate any evidence of market anomalies D. joint tests of market efficiency and the risk-adjustment measure

D

One of the main problems with the arbitrage pricing theory is ________. A) its use of several factors instead of a single market index to explain the risk-return relationship B) the introduction of nonsystematic risk as a key factor in the risk-return relationship C) that the APT requires an even larger number of unrealistic assumptions than does the CAPM D) the model fails to identify the key macroeconomic variables in the risk-return relationship

D

Privately held firms may have only ________ shareholders. A) 10 B) 99 C) 250 D) 2,000

D

Proponents of the EMH think technical analysts __________. A. should focus on relative strength B. should focus on resistance levels C. should focus on support levels D. are wasting their time

D

Random price movements indicate ________. A. irrational markets B. that prices cannot equal fundamental values C. that technical analysis to uncover trends can be quite useful D. that markets are functioning efficiently

D

Rank the following types of markets from least integrated and organized to most integrated and organized: I. Brokered markets II. Continuous auction markets III. Dealer markets IV. Direct search markets A) IV, II, I, III B) I, III, IV, II C) II, III, IV, I D) IV, I, III, II

D

Restrictions on trading involving insider information apply to: I. Corporate officers and directors II. Major stockholders III. Relatives of corporate directors and officers A) I only B) I and II only C) II and III only D) I, II, and III

D

Risk that can be eliminated through diversification is called ________ risk. A) unique B) firm-specific C) diversifiable D) all of these options

D

Someone who invests in the Vanguard Index 500 mutual fund could most accurately be described as using which approach? A. Active management B. Arbitrage C. Fundamental analysis D. Passive investment

D

Suppose that in 2018 the expected dividends of the stocks in a broad market index equaled $240 million when the discount rate was 8% and the expected growth rate of the dividends equaled 6%. Using the constant-growth formula for valuation, if interest rates increase to 9%, the value of the market will change by ________. A) -10% B) -20% C) -25% D) -33%

D

TIPS are ________. A) Treasury bonds that pay no interest and are sold at a discount B) U.K. bonds that protect investors from default risk C) securities that trade on the Toronto stock index D) Treasury bonds that protect investors from inflation

D

The CAPM ________. A) predicts the relationship between risk and expected return of an asset B) provides a benchmark rate of return for evaluating possible investments C) helps us make an educated guess as to expected return on assets that have not yet traded in the marketplace D) All of the options.

D

The CFA Institute Standards of Professional Conduct require that members ________. A) place their clients' interests before their own B) disclose conflicts of interest to clients C) inform their employers that they are obligated to comply with the Standards of Professional Conduct D) all of these options

D

The Hang Seng index reflects market performance on which of the following major stock markets? A) Japan B) Singapore C) Taiwan D) Hong Kong

D

The May 17, 2015, price quotation for a Boring call option with a strike price of $50 due to expire in November is $20.80, while the stock price of Boring is $69.80. The premium on one Boring November 50 call contract is ________. A) $1,980 B) $4,900 C) $5,000 D) $2,080

D

The NYSE Hybrid Market allows ________. A) individuals to send orders directly to a specialist B) individuals to send orders directly to an electronic system C) brokers to send orders directly to a specialist D) brokers to send orders either to an electronic system or to a specialist

D

The SIPC was established by the ________. A) Insider Trading Act of 1931 B) Securities Act of 1933 C) Securities Exchange Act of 1934 D) none of these options

D

The SML is valid for ________, and the CML is valid for ________. A) only individual assets; well-diversified portfolios only B) only well-diversified portfolios; only individual assets C) both well-diversified portfolios and individual assets; both well-diversified portfolios and individual assets D) both well-diversified portfolios and individual assets; well-diversified portfolios only

D

The ________ system enables exchange members to send orders directly to a specialist over computer lines. A) FAX B) Direct Plus C) NASDAQ D) SUPERDOT

D

The bonds of Elbow Grease Dishwashing Company have received a rating of C by Moody's. The C rating indicates that the bonds are ________. A) high grade B) intermediate grade C) investment grade D) junk bonds

D

The commission structure on a stock purchase is $20 plus $.02 per share. If you purchase four round lots of a stock selling for $56, what is your commission? A) $20 B) $22 C) $26 D) $28

D

The commission structure on a stock purchase is $50 plus $.03 per share. If you purchase 600 shares of a stock selling for $65, what is your commission? A) $35 B) $45 C) $53 D) $68

D

The common stock of the Avalon Corporation has been trading in a narrow range around $40 per share for months, and you believe it is going to stay in that range for the next 3 months. The price of a 3-month put option with an exercise price of $40 is $3, and a call with the same expiration date and exercise price sells for $4. Selling a straddle would generate total premium income of ________. A) $300 B) $400 C) $500 D) $700

D

The correlation coefficient between two assets equals ________. A) their covariance divided by the product of their variances B) the product of their variances divided by their covariance C) the sum of their expected returns divided by their covariance D) their covariance divided by the product of their standard deviations

D

The cost of buying and selling a stock includes: I. Broker's commissions II. Dealer's bid-asked spread III. Price concessions that investors may be forced to make A) I and II only B) II and III only C) I and III only D) I, II, and III

D

The dollar-weighted return is the ________. A) difference between cash inflows and cash outflows B) arithmetic average return C) geometric average return D) internal rate of return

D

The expected return of the risky-asset portfolio with minimum variance is ________. A) the market rate of return B) zero C) the risk-free rate D) The answer cannot be determined from the information given.

D

The interest rate charged by large banks in London to lend money among themselves is called ________. A) the prime rate B) the discount rate C) the federal funds rate D) LIBOR

D

The market portfolio has a beta of ________. A) -1 B) 0 C) .5 D) 1

D

The market share held by the NYSE Arca system in February 2011 was approximately ________. A) 65% B) 45% C) 25% D) 10%

D

The measure of risk used in the capital asset pricing model is ________. A) specific risk B) the standard deviation of returns C) reinvestment risk D) beta

D

The minimum tick size, or spread between prices in the Treasury bond market, is A) 1/8 of a point. B) 1/16 of a point. C) 1/32 of a point. D) 1/128 of a point.

D

The non-European country with the highest average first-day returns in 2014 was ________. A) Canada B) United States C) China D) Jordan

D

The return on the risky portfolio is 15%. The risk-free rate, as well as the investor's borrowing rate, is 10%. The standard deviation of return on the risky portfolio is 20%. If the standard deviation on the complete portfolio is 25%, the expected return on the complete portfolio is ________. A) 6% B) 8.75 % C) 10% D) 16.25%

D

The semistrong form of the efficient market hypothesis implies that ____________ generate abnormal returns and ____________ generate abnormal returns. A. technical analysis cannot; fundamental analysis can B. technical analysis can; fundamental analysis can C. technical analysis can; fundamental analysis cannot D. technical analysis cannot; fundamental analysis cannot

D

The two-factor model on a stock provides a risk premium for exposure to market risk of 12%, a risk premium for exposure to silver commodity prices of 3.5%, and a risk-free rate of 4%. The beta for exposure to market risk is 1, and the beta for exposure to commodity prices is also 1. What is the expected return on the stock? A) 11.6% B) 13% C) 15.3% D) 19.5%

D

The yield on tax-exempt bonds is ________. A) usually less than 50% of the yield on taxable bonds B) normally about 90% of the yield on taxable bonds C) greater than the yield on taxable bonds D) less than the yield on taxable bonds

D

The yield to maturity on a bond is: I. Above the coupon rate when the bond sells at a discount and below the coupon rate when the bond sells at a premium II. The discount rate that will set the present value of the payments equal to the bond price III. Equal to the true compound return on investment only if all interest payments received are reinvested at the yield to maturity A) I only B) II only C) I and II only D) I, II, and III

D

There are two independent economic factors, M1 and M2. The risk-free rate is 5%, and all stocks have independent firm-specific components with a standard deviation of 25%. Portfolios A and B are well diversified. Given the data below, which equation provides the correct pricing model? Portfolio Beta on M1 Beta on M2 E[rp] A 1.5 1.75 35% B 1.0 0.65 20% A) E(rP) = 5 + 1.12βP1 + 11.86βP2 B) E(rP) = 5 + 4.96βP1 + 13.26βP2 C) E(rP) = 5 + 3.23βP1 + 8.46βP2 D) E(rP) = 5 + 8.71βP1 + 9.68βP2

D

Todd Mountain Development Corporation is expected to pay a dividend of $3 in the upcoming year. Dividends are expected to grow at the rate of 8% per year. The risk-free rate of return is 5%, and the expected return on the market portfolio is 17%. The stock of Todd Mountain Development Corporation has a beta of 0.75. Using the constant-growth DDM, the intrinsic value of the stock is ________. A) $4 B) $17.65 C) $37.50 D) $50

D

Trading on inside information is: I. Prohibited by federal law II. Prohibited by the CFA Institute Standards of Professional Conduct III. Monitored by the SEC A) I and II only B) II and III only C) I and III only D) I, II, and III

D

Treasury notes have initial maturities between ________ years. A) 2 and 4 B) 5 and 10 C) 10 and 30 D) 1 and 10

D

Under firm-commitment underwriting, the ________ assumes the full risk that the shares cannot be sold to the public at the stipulated offering price. A) red herring B) issuing company C) initial stockholder D) underwriter

D

Warrants differ from listed options in that: I. Exercise of warrants results in dilution of a firm's earnings per share. II. When warrants are exercised, new shares of stock must be created. III. Warrant exercise results in cash flows to the firm, whereas exercise of listed options does not. A) I only B) I and II only C) II and III only D) I, II, and III

D

What happened to the effective spread on trades when the SEC allowed the minimum tick size to move from one-eighth of a dollar to one-sixteenth of a dollar in 1997 and from one-sixteenth of a dollar to one cent in 2001? A) The effective spread increased in 1997 but decreased in 2001. B) The effective spread increased in both cases. C) The effective spread decreased in 1997 but increased in 2001. D) The effective spread decreased in both cases.

D

What is the VaR of a $10 million portfolio with normally distributed returns at the 5% VaR? Assume the expected return is 13% and the standard deviation is 20%. A) 13% B) -13% C) 19.90% D) -19.90%

D

When all investors analyze securities in the same way and share the same economic view of the world, we say they have ________. A) heterogeneous expectations B) equal risk aversion C) asymmetric information D) homogeneous expectations

D

When matching orders from the public, a specialist is required to use the ________. A) lowest outstanding bid price and highest outstanding ask price B) highest outstanding bid price and highest outstanding ask price C) lowest outstanding bid price and lowest outstanding ask price D) highest outstanding bid price and lowest outstanding ask price

D

Which of the following beliefs would not preclude charting as a method of portfolio management? A. The market is strong-form efficient. B. The market is semistrong-form efficient. C. The market is weak-form efficient. D. Stock prices follow recurring patterns.

D

Which of the following correlation coefficients will produce the least diversification benefit? A) -.6 B) -.3 C) 0 D) .8

D

Which of the following indexes are market value-weighted? I. The NYSE Composite II. The S&P 500 III.The Wilshire 5000 A) I and II only B) II and III only C) I and III only D) I, II, and III

D

Which of the following is (are) true about dark pools? I. They allow anonymity in trading. II. They often involve large blocks of stocks. III. Trades made through them might not be reported. A) I and II only B) II and III only C) I and III only D) I, II, and III

D

Which of the following is a correct expression concerning the formula for the standard deviation of returns of a two-asset portfolio where the correlation coefficient is positive? A) σ2rp < (W12σ12 + W22σ22) B) σ2rp = (W12σ12 + W22σ22) C) σ2rp = (W12σ12 - W22σ22) D) σ2rp > (W12σ12 + W22σ22)

D

Which of the following is not a characteristic of a money market instrument? A) liquidity B) marketability C) low risk D) maturity greater than 1 year

D

Which of the following is not considered a money market investment? A) bankers' acceptance B) eurodollar C) repurchase agreement D) Treasury note

D

Which of the following reforms were not included in 2014 regulations regarding money market funds? A) Institutional funds will "float" the prices of their shares. B) Funds can limit redemptions or impose a 2% fee if assets fall by more than 30%. C) increased disclosure of assets' values and liquidity D) All of the options were included.

D

Which of the following statements about convertible bonds are true? I. The conversion price does not change over time. II. The associated stocks may not pay dividends as long as the bonds are outstanding. III. Most convertibles are also callable at the discretion of the firm. IV. They may be thought of as straight bonds plus a call option. A) I and III only B) I and IV only C) I, II, and IV only D) III and IV only

D

Which of the following statements is (are) correct? A. If a market is weak-form efficient, it is also semistrong- and strong-form efficient. B. If a market is semistrong-form efficient, it is also strong-form efficient. C. If a market is strong-form efficient, it is also semistrong- but not weak-form efficient. D. If a market is strong-form efficient, it is also semistrong- and weak-form efficient.

D

Which of the following strategies makes a profit when the stock price declines and loses money when the stock price increases? A) long call and short put B) long call and long put C) short call and short put D) short call and long put

D

Which of the following valuation measures is often used to compare firms that have no earnings? A) price-to-book ratio B) P/E ratio C) price-to-cash-flow ratio D) price-to-sales ratio

D

Which one of the following is the ticker symbol for the CBOE option contract on the S&P 100 Index? A) SPX B) DJX C) CME D) OEX

D

Which one of the following statements about market and book value is correct? A) All firms sell at a market-to-book ratio above 1. B) All firms sell at a market-to-book ratio greater than or equal to 1. C) All firms sell at a market-to-book ratio below 1. D) Most firms have a market-to-book ratio above 1, but not all.

D

Which one of the following would be considered a risk-free asset in real terms as opposed to nominal? A) money market fund B) U.S. T-bill C) short-term corporate bonds D) U.S. T-bill whose return was indexed to inflation

D

Which one of the statements about margin requirements on option positions is not correct? A) The margin required will be lower if the option is in the money. B) If the required margin exceeds the posted margin, the option writer will receive a margin call. C) A buyer of a put or call option does not have to post margin. D) Even if the writer of a call option owns the stock, the writer will have to meet the margin requirement in cash.

D

Which risk can be partially or fully diversified away as additional securities are added to a portfolio? I. Total risk II. Systematic risk III. Firm-specific risk A) I only B) I and II only C) I, II, and III D) I and III

D

You are cautiously bullish on the common stock of the Wildwood Corporation over the next several months. The current price of the stock is $50 per share. You want to establish a bullish money spread to help limit the cost of your option position. You find the following option quotes: Wildwood Corp Underlying Stock price: $50.00 Expiration Strike Call Put June 45.00 8.50 2.00 June 50.00 4.50 3.00 June 55.00 2.00 7.50 Ignoring commissions, the cost to establish the bull money spread with calls would be ________. A) $1,050 B) $650 C) $400 D) $400 income rather than cost

D

You are cautiously bullish on the common stock of the Wildwood Corporation over the next several months. The current price of the stock is $50 per share. You want to establish a bullish money spread to help limit the cost of your option position. You find the following option quotes: Wildwood Corp Underlying Stock price: $50.00 Expiration Strike Call Put June 45.00 8.50 2.00 June 50.00 4.50 3.00 June 55.00 2.00 7.50 Suppose you establish a bullish money spread with the puts. In June the stock's price turns out to be $52. Ignoring commissions, the net profit on your position is ________. A) $500 B) $700 C) $200 D) $250

D

You are considering adding a new security to your portfolio. To decide whether you should add the security, you need to know the security's: I. Expected return II. Standard deviation III. Correlation with your portfolio A) I only B) I and II only C) I and III only D) I, II, and III

D

You buy a bond with a $1,000 par value today for a price of $875. The bond has 6 years to maturity and makes annual coupon payments of $75 per year. You hold the bond to maturity, but you do not reinvest any of your coupons. What was your effective EAR over the holding period? A) 10.4% B) 9.57% C) 7.45% D) 8.78%

D

You buy a call option on Merritt Corp. with an exercise price of $50 and an expiration date in July, and you write a call option on Merritt Corp. with an exercise price of $55 and an expiration date in July. This is called a ________. A) time spread B) long straddle C) short straddle D) money spread

D

You find that the bid and ask prices for a stock are $10.25 and $10.30, respectively. If you purchase or sell the stock, you must pay a flat commission of $25. If you buy 100 shares of the stock and immediately sell them, what is your total implied and actual transaction cost in dollars? A) $50 B) $25 C) $30 D) $55

D

You have $500,000 available to invest. The risk-free rate, as well as your borrowing rate, is 8%. The return on the risky portfolio is 16%. If you wish to earn a 22% return, you should ________. A) invest $125,000 in the risk-free asset B) invest $375,000 in the risk-free asset C) borrow $125,000 D) borrow $375,000

D

You have the following rates of return for a risky portfolio for several recent years: 2013 35.23% 2014 18.67% 2015 −9.87% 2016 23.45% The annualized (geometric) average return on this investment is ________. A) 16.15% B) 16.87% C) 21.32% D) 15.60%

D

You own $75,000 worth of stock, and you are worried the price may fall by year-end in 6 months. You are considering using either puts or calls to hedge this position. Given this, which of the following statements is (are) correct? I. One way to hedge your position would be to buy puts. II. One way to hedge your position would be to write calls. III. If major stock price declines are likely, hedging with puts is probably better than hedging with short calls. A) I only B) II only C) I and III only D) I, II, and III

D

You purchase one MBI July 125 call contract (equaling 100 shares) for a premium of $5. You hold the option until the expiration date, when MBI stock sells for $123 per share. You will realize a ________ on the investment. A) $200 profit B) $200 loss C) $500 profit D) $500 loss

D

You purchase one MBI July 90 call contract for a premium of $4. The stock has a 2-for-1 split prior to the expiration date. You hold the option until the expiration date, when MBI stock sells for $48 per share. You will realize a ________ on the investment. A) $300 profit B) $100 loss C) $400 loss D) $200 profit

D

You purchased XYZ stock at $50 per share. The stock is currently selling at $65. Your gains could be protected by placing a ________. A) limit buy order B) limit sell order C) market order D) stop-loss order

D

You put up $50 at the beginning of the year for an investment. The value of the investment grows 4% and you earn a dividend of $3.50. Your HPR was ________. A) 4% B) 3.5% C) 7% D) 11%

D

You short-sell 200 shares of Tuckerton Trading Co., now selling for $50 per share. What is your maximum possible loss? A) $50 B) $150 C) $10,000 D) unlimited

D

You would typically find all but which one of the following in a bond contract? A) a dividend restriction clause B) a sinking fund clause C) a requirement to subordinate any new debt issued D) a price-earnings ratio

D

Your investment has a 20% chance of earning a 30% rate of return, a 50% chance of earning a 10% rate of return, and a 30% chance of losing 6%. What is your expected return on this investment? A) 12.8% B) 11% C) 8.9% D) 9.2%

D

______ option can only be exercised on the expiration date. A) A Mexican B) An Asian C) An American D) A European

D


Ensembles d'études connexes

Academic Success, Compare and Contrast Graphic Organizers Practice- 100%

View Set

History: Empire Builders: Chaldeans

View Set

random for module 2 exam off of EAQ questions

View Set

Transformation and Gene Mutations

View Set

Alternative Energy Day 4 Paxton Patterson

View Set

BSA/AML Complying with BSA, GLOSSARY BSA_AML_Money Laundering RED FLAGS, BSA/AML Complying with BSA

View Set